Quiz 4

¡Supera tus tareas y exámenes ahora con Quizwiz!

A family physician is performing patient teaching about the influenza virus with each patient who has come to the clinic to receive that year's vaccine. Which of the following statements by the patient best reflects an accurate understanding of the flu virus? A) "I could come down with viral or bacterial pneumonia as a result of a bad flu bug." B) "I know my vaccination is especially important since there aren't any drugs that can treat the flu once I get sick with it." C) "The emphasis on bundling up, staying warm, and drinking lots of fluids is outdated and actually ineffective." D) "Like all vaccines, it is ideal if everyone in a population gets immunized against the flu."

A) "I could come down with viral or bacterial pneumonia as a result of a bad flu bug." Viral and bacterial pneumonia are known sequelae of influenza. Antiviral drugs do exist for the flu, and the efficacy of staying warm and increasing fluid consumption have been demonstrated. The flu vaccine is recommended for higher risk individuals, and guidelines do not indicate the need for all individuals to be vaccinated.

The mother of a 7-year-old boy who has recently been diagnosed with childhood asthma has come to the education center to learn more about her son's condition. Which of the following teaching points is most justifiable? A) "Research has shown that viruses may actually be a factor in many children's asthma." B) "The most reliable indicator that your child is having an asthma attack is audible wheezing." C) "Steroids that your child can inhale will likely be the first line of defense." D) "Your son will likely need to limit or avoid exercise and sports."

A) "Research has shown that viruses may actually be a factor in many children's asthma." Viruses have been implicated as a contributing factor in childhood asthma. Wheezing may or may not be present in children, and inhaled corticosteroids are not common as an initial therapy. Current treatment guidelines do not advise the categorical avoidance of exercise.

During a prenatal education class, a participant has related a story about how her friend's infant died of sudden infant death syndrome (SIDS). What can the educator tell the group about how they can prevent SIDS when they have their babies? Select all that apply. A) "The best sleeping position for your baby is on his back." B) "Children are at particular risk of SIDS when they have a cold or flu, so these times require extra vigilance." C) "Using drugs during pregnancy has been shown to be associated with SIDS after birth, which is one more reason for mothers to avoid them." D) "It's important if anyone in your home smokes to make sure they only do it outside." E) "The exact cause of SIDS still isn't known, so there's little that you can do to prevent this tragic event."

A) "The best sleeping position for your baby is on his back." C) "Using drugs during pregnancy has been shown to be associated with SIDS after birth, which is one more reason for mothers to avoid them." D) "It's important if anyone in your home smokes to make sure they only do it outside." Prone or side-lying position, intrauterine drug exposure, and postnatal exposure to cigarette smoke are all associated with SIDS. Upper respiratory infections are not noted to present a particular risk, and though the exact etiology is not known, preventative measures do exist

A midwife who is providing care for a woman during her first pregnancy is assessing for intrauterine growth retardation (IUGR) during an early prenatal checkup. Which of the following questions best addresses the risks for IUGR? A) "What does your typical diet look like over the course of a day?" B) "What is the highest level of education that you've finished?" C) "Are there many people in your life that you can count on for help and support?" D) "How would you describe your mood since you've been pregnant?"

A) "What does your typical diet look like over the course of a day?" Nutrition is a key aspect in the prevention of IUGR. Educational level, the presence or absence of support systems, and psychosocial health may all have ramifications that could affect fetal development, but they have a less direct bearing than maternal nutrition.

The first-time parents of an infant girl 2 days postpartum are distressed at the jaundiced appearance of her skin and are eager for both an explanation and treatment for the problem. Which of the following responses by their physician is most accurate? A) "Your daughter's young liver is unable to get rid of the waste products from old red blood cells." B) "Because your daughter's kidneys are so small, they have a hard time getting rid of the wastes that are always accumulating in her blood." C) "Nearly half of all infants have this problem, and while it is distressing to look at, it is largely harmless and will resolve in time." D) "This is a sign that your baby needs more milk than she is currently getting, and increased breast-feeding will act to flush these pigments out of her system."

A) "Your daughter's young liver is unable to get rid of the waste products from old red blood cells." Bilirubin is formed from the breakdown of hemoglobin in red blood cells. Normally about two thirds of the unconjugated bilirubin produced by a term newborn can be effectively cleared by the liver. However, the relative immaturity of the newborn liver and the shortened life span of the fetal red blood cells may predispose the term newborn to hyperbilirubinemia. Bilirubin clearance is not the domain of the kidneys, and treatment is often necessary. Jaundice can sometimes be addressed by increasing breast-feeding, but it is not a sign in and of itself of insufficient feeding.

Which of the following patients who presented to a walk-in medical clinic is most likely to be diagnosed with a rhinosinusitis rather than a common cold? A) A man complaining of general fatigue, a headache, and facial pain with a temperature of 100.9°F B) A woman presenting with malaise, lethargy, and copious nasal secretions C) A man with a dry, stuffy nasopharynx, a sore throat, and temperature of 98.9°F D) A woman complaining of generalized aches and who has a hoarse voice and reddened, painful upper airways

A) A man complaining of general fatigue, a headache, and facial pain with a temperature of 100.9°F Fever and facial pain are more commonly associated with rhinosinusitis rather than the common cold. The other noted symptoms are indicative of the common cold rather than rhinosinusitis.

The neonatologist suspects an infant has developed sepsis with multiorgan system illness. The nurse caring for this infant will note which of the assessment findings support this diagnosis. Select all that apply. A) Decreasing BP with increase in heart rate indicative of shock B) Prolonged PT and PTT and decrease in platelet count C) Frequent voiding of a small amount of light-colored urine D) Bilateral warm feet but pedal pulses hard to palpate E) Positive Moro reflex when loud noise made at crib side

A) Decreasing BP with increase in heart rate indicative of shock B) Prolonged PT and PTT and decrease in platelet count Premature infants' health is severely impacted by early-onset infections and progressive multiorgan system illness. Infants with sepsis frequently present with respiratory failure, shock, meningitis, DIC, acute tubular necrosis, and symmetrical peripheral gangrene. Positive Moro reflex is normal for this infant.

A premature infant who is receiving care in a neonatal intensive care unit (NICU) has just been identified as having necrotizing enterocolitis (NEC). Of the following clinical manifestations, identify those most likely to contribute to the diagnosis of NEC. Select all that apply. A) Feeding intolerance B) Inability to pass stool within the first 10 days of life C) Hard, taut abdomen with increasing distention D) Blood noted in stools E) Hypoactive bowel sounds on right lower quadrant

A) Feeding intolerance C) Hard, taut abdomen with increasing distention D) Blood noted in stools Immature immunity, shunting of circulation away from the GI tract, and infectious processes have all been implicated in the etiology of NEC. The classic initial symptoms are usually feeding intolerance, abdominal distention, and bloody stools shortly after the first week of life.

A premature infant on mechanical ventilation has developed bronchopulmonary dysplasia (BPD) and is showing signs and symptoms of hypoxemia, low lung compliance, and respiratory distress. Which of the following is the most likely contributor to the infant's present health problem? A) High-inspired oxygen concentration and injury from positive-pressure ventilation B) Failure to administer corticosteroids to the infant in utero C) Insufficient surfactant production and insufficient surfactant therapy D) Insufficient supplemental oxygen therapy

A) High-inspired oxygen concentration and injury from positive-pressure ventilation Despite the administration of corticosteroids in utero to hasten alveolar maturation, premature infants suffering respiratory distress syndrome often must be treated with supplemental oxygen and mechanical ventilation. However, overly forceful positive-pressure ventilation (barotrauma) can lead to the chronic lung impairment of BPD. Surfactant therapy is a first-line defense against the development of RDS and is also used to treat cases of BPD; additional time on a ventilator is often required as well.

. A nurse is providing care for a client who has been admitted to a medical unit with a diagnosis of bronchiectasis. Which of the following signs and symptoms should the nurse expect to find during physical assessment of the client and the review of the client's history? Select all that apply. A) Recurrent chest infections B) Production of purulent sputum C) A barrel chest D) Low hemoglobin levels E) Recent surgery

A) Recurrent chest infections B) Production of purulent sputum D) Low hemoglobin levels Chest infections, copious production of purulent sputum, and anemia are all associated with bronchiectasis. A barrel chest is more commonly evident with emphysema, and recent surgery is not a noted factor.

A patient arrives in the ED after an automobile accident. Which of the following clinical manifestations lead the nurse to suspect a pneumothorax? Select all that apply. A) Respiratory rate 34 B) Asymmetrical chest movements, especially on inspiration C) Diminished breath sounds over the painful chest area D) Pulse oximetry 98% E) ABG pH level of 7.38

A) Respiratory rate 34 B) Asymmetrical chest movements, especially on inspiration C) Diminished breath sounds over the painful chest area Manifestations of pneumothorax include increase in respiratory rate, dyspnea, asymmetrical movements of the chest wall, especially during inspiration, hyperresonant sound on percussion, and decreased or absent breath sounds over the area of pneumothorax. The pulse oximetry reading is normal. ABG pH level of 7.38 is a normal finding

A COPD patient asks the nurse what medications are prescribed to help his breathing. The nurse, looking at the list of medications, will educate the patient about which of the following medications to help his COPD in the long term? Select all that apply. A) Salmeterol (Serevent), a bronchodilator B) Tiotropium (Spiriva), anticholinergic C) Alprazolam (Xanax), a benzodiazepine D) Sildenafil (Viagra), a vasodilator E) Ketorolac (Toradol), an NSAID

A) Salmeterol (Serevent), a bronchodilator B) Tiotropium (Spiriva), anticholinergic Pharmacologic treatment of COPD includes the use of bronchodilators (Serevent) and anticholinergic drugs (Tiotropium). Benzodiazepines are used for anxiety, and sildenafil is a vasodilator commonly prescribed not only for erectile dysfunction but also for patients with pulmonary hypertension. Toradol (ketorolac) is an NSAID for pain and inflammation.

The nurse caring for a male child with respiratory problems is concerned he may be developing respiratory failure. Upon assessment, the nurse knows that which of the following are clinical manifestations of respiratory failure? Select all that apply. A) Severe accessory muscle retractions B) Nasal flaring C) Grunting on expiration D) Inspiratory wheezes heard E) Swollen glottis

A) Severe accessory muscle retractions B) Nasal flaring C) Grunting on expiration Children with impending respiratory failure due to airway or lung disease have rapid breathing; exaggerated use of the accessory muscles; retractions, which are more pronounced in the child than in an adult because of more compliant chest; nasal flaring; and grunting during expiration. Inspiratory wheezes are usually associated with asthma. Swollen glottis can occur with strep throat.

A patient with pulmonary hypertension may display which of the following clinical manifestations? Select all that apply. A) Shortness of breath B) Decreased exercise tolerance C) Nasal flaring D) Grunting on expiration E) Swelling (edema) of the legs and feet

A) Shortness of breath B) Decreased exercise tolerance E) Swelling (edema) of the legs and feet Symptoms of PAH typically progress from shortness of breath and decreasing exercise tolerance to right heart failure, with marked peripheral edema and functional limitations. Other common symptoms include fatigue, angina, and syncope (fainting) or near-syncope. Nasal flaring and expiratory grunting are usually seen in infants experiencing respiratory distress.

A nurse is providing care for a patient who has been admitted with a newly diagnosed bilateral pleural effusion. Which of the following findings from the nurse's initial assessment of the patient is incongruent with the patient's diagnosis and would require further investigation? A) The client complains of sharp pain exacerbated by deep inspiration. B) The client's breath sounds are diminished on auscultation. C) Pulse oximetry indicates that the client is hypoxemic. D) The client complains of dyspnea and increased work of breathing.

A) The client complains of sharp pain exacerbated by deep inspiration. Pleural effusion is not normally associated with pain, and intense pain that is worsened by deep breathing would necessitate further investigation. Diminished breath sounds, hypoxemia, and dyspnea are common findings associated with pleural effusion.

A patient with small cell lung cancer (SCLC) has developed a paraneoplastic syndrome called Cushing syndrome. Based on this new complication, the nurse will likely assess which of the following clinical manifestations of Cushing syndrome? A) Weight gain, moon face, buffalo hump, and purple striae on the abdomen B) Bilateral edema in the arms, swollen face, and protruding eyes C) Severe bone/joint pain, nausea/vomiting, and polyuria D) Tetany, new-onset seizure activity, emotional lability, and extrapyramidal symptoms

A) Weight gain, moon face, buffalo hump, and purple striae on the abdomen SCLS is associated with several types of paraneoplastic syndromes, including Cushing's. Answer choice B refers to superior vena cava syndrome; answer choice C refers to hypercalcemia; and answer choice D refers to tumor lysis syndrome. All of these are complications that can occur with cancer and treatment of cancer.

Which of the following clients are displaying known risk factors for the development of pulmonary emboli? Select all that apply. A client who is: A) immobilized following orthopedic surgery. B) experiencing impaired Cl- and Na+ regulation. C) taking amiodarone for the treatment of premature ventricular contractions. D) a smoker and who takes oral contraceptives. E) undergoing radiation therapy for the treatment of breast cancer.

A) immobilized following orthopedic surgery. D) a smoker and who takes oral contraceptives. Postsurgical immobility, smoking, and the use of oral contraceptives are all identified risk factors for the development of pulmonary emboli. Impaired Cl- and Na+ regulation are associated with cystic fibrosis, while amiodarone and radiation therapy are linked to interstitial lung diseases.

A patient who has been on a high-protein diet comes to the emergency department with respiratory symptoms. Upon analysis of arterial blood gases (ABGs), the patient is diagnosed with hypercapnia. The nurse will note the ABG results that confirm this diagnosis include: Select all that apply. A) pH 7.31 (normal 7.35 to 7.45). B) PO2 of 97%. C) PCO2 of 58 mm Hg (normal 38 to 42). D) Serum HCO3of -33 mEq/L (normal 22 to 28). E) Serum K+ (potassium)of 3.6 mmol/L (normal 3.5 to 5.0).

A) pH 7.31 (normal 7.35 to 7.45). C) PCO2 of 58 mm Hg (normal 38 to 42). D) Serum HCO3of -33 mEq/L (normal 22 to 28). Hypercapnia affects a number of body functions, including acid-base balance and renal, neurological, and CV functions. Elevated levels of PCO2 (38 to 42) produce a decrease in pH (7.35 to 7.45) and respiratory acidosis. Compensatory mechanisms result in an increase in serum HCO3 (22 to 28). In this example, the PO2 level is within normal range. Serum K+ is not part of the ABG analysis.

A 62-year-old female smoker is distraught at her recent diagnosis of small cell lung cancer (SCLC). How can her physician most appropriately respond to her? A) "I'm sure this is very hard news to hear, but be aware that with aggressive treatment, your chances of beating this are quite good." B) "This is very difficult to hear, I'm sure, and we have to observe to see if it spreads because that often happens." C) "I'm very sorry to have to give you this news; I'd like to talk to you about surgical options, however." D) "This is a difficult diagnosis to receive, but there is a chance that the cancer may go into remission."

B) "This is very difficult to hear, I'm sure, and we have to observe to see if it spreads because that often happens." Metastases are common with SCLC. Survival rates are very low; surgical options do not exist; and remission is very unlikely.

A 51-year-old female client who is 2 days postoperative in a surgical unit of a hospital is at risk of developing atelectasis as a result of being largely immobile. Which of the following teaching points by her nurse is most appropriate? A) "Being in bed increases the risk of fluid accumulating between your lungs and their lining, so it's important for you to change positions often." B) "You should breathe deeply and cough to help your lungs expand as much as possible while you're in bed." C) "Make sure that you stay hydrated and walk as soon as possible to avoid us having to insert a chest tube." D) "I'll proscribe bronchodilator medications that will help open up your airways and allow more oxygen in."

B) "You should breathe deeply and cough to help your lungs expand as much as possible while you're in bed." Atelectasis is characterized by incomplete lung expansion and can often be prevented by deep breathing and coughing. Pleural effusion, not atelectasis, is associated with fluid accumulation between the lungs and their lining, and neither chest tube insertion nor bronchodilators are common treatments for atelectasis.

Upon admission to the ICU, a patient with a history of cor pulmonale will likely be exhibiting which of the following clinical manifestations of right heart failure? Select all that apply. A) Fine crackles throughout both lung fields B) +4 pitting edema in lower extremities C) Expectorating copious amounts of frothy, pink sputum D) Jugular vein distension E) Altered level of consciousness

B) +4 pitting edema in lower extremities D) Jugular vein distension E) Altered level of consciousness Signs of right-sided HF include venous congestion (jugular vein distension), peripheral edema (+4 pitting edema in feet), shortness of breath, and productive cough. Altered level of consciousness may occur as the result of carbon dioxide retention. Fine crackles in all lung fields and frothy, pink sputum are common in left-sided HF.

Which of the following phenomena is most likely occurring during a child's alveolar stage of lung development? A) Terminal alveolar sacs are developing, and surfactant production is beginning. B) A single capillary network exists, and the lungs are capable of respiration. C) The conducting airways are formed, but respiration is not yet possible. D) Primitive alveoli are formed, and the bronchi and bronchioles become much larger.

B) A single capillary network exists, and the lungs are capable of respiration. During the alveolar stage of lung development from late fetal to early childhood, a single capillary network appears, and the lungs are ready to perform respiration. The development of alveolar sacs and production of surfactant are associated with the saccular period, and formation of the conducting airways occurs during the pseudoglandular period. Formation of primitive alveoli takes place during the canalicular period.

A 41-year-old male client has presented to the emergency department with an acute onset of increased respiratory rate and difficulty breathing. STAT chest x-ray indicates diffuse bilateral infiltrates of his lung tissue, and ECG displays no cardiac dysfunction. What is this client's most likely diagnosis? A) Cor pulmonale B) Acute lung injury C) Pulmonary hypertension D) Sarcoidosis

B) Acute lung injury Rapid onset of respiratory distress accompanied by diffuse bilateral infiltrates of lung tissue and an absence of cardiac changes are associated with acute lung injury/acute respiratory distress syndrome. These particular signs and symptoms are not as closely associated with cor pulmonale, pulmonary hypertension, or sarcoidosis.

As a consequence of a long-standing lung disease, a client is in a chronic state of hypoxia. Which of the following phenomena would the client's care team be most justified in anticipating? Select all that apply. A) Metabolic alkalosis B) Increased erythropoietin production C) Pulmonary vasodilation D) Hyperventilation E) Personality changes

B) Increased erythropoietin production D) Hyperventilation E) Personality changes Increased production of erythropoietin, hyperventilation, and cognitive and personality changes are all associated with hypoxemia. Acidosis, not alkalosis, and vasoconstriction rather than vasodilation are likely to occur.

The nursery has just admitted a new infant born 1 hour ago. While performing an assessment, the nurse suspects the infant may have hypoglycemia based on which of the following assessment data? Select all that apply. A) Heel stick glucose value of 50 mg/dL B) Infant having periods of apnea requiring physical stimulation C) Muscle twitching noted while lying in crib undisturbed by nurses D) Hyperactive reflexes noted especially when crying E) Poor suck reflex resulting in an inability to feed properly

B) Infant having periods of apnea requiring physical stimulation C) Muscle twitching noted while lying in crib undisturbed by nurses E) Poor suck reflex resulting in an inability to feed properly In neonates, glucose levels stabilize to a value of 50 mg/dL or higher within the first 3 hours of life. Concentrations below 45 mg/dL should be considered abnormal. Signs and symptoms of neonatal hypoglycemia include cyanosis, apnea, hypothermia, hypotonia, poor feeding, lethargy, and seizures.

A nurse who works in a neonatal intensive care unit is providing care for an infant born at 26 weeks' gestation. Which of the following assessments would lead the nurse to suspect that the infant has developed respiratory distress syndrome (RDS)? A) The infant's blood pressure and temperature are normal measurements as expected. B) Infant is grunting and has notable intercostal retractions with respirations. C) Infant has poor motor skills and limited limb range of motion. D) Infant has apnea lasting 5 to 10 seconds with a decrease in heart rate, which reverses with tactile stimulation.

B) Infant is grunting and has notable intercostal retractions with respirations. While premature birth is associated with numerous potential health problems, including variations in vital signs, impaired motor function, and neurological deficits, the most common complications of prematurity involve respiratory function

What topic should health promotion initiatives emphasize if the target audience is parents of preschoolers and the goal is to minimize mortality? A) Handwashing as an infection control measure B) Injury prevention especially when the child is near water C) Identifying signs of child abuse and neglect D) The importance of good nutrition

B) Injury prevention especially when the child is near water Injuries are the leading cause of death in children aged 1 to 4. While handwashing does prevent many infections, these are not commonly fatal. Likewise, child abuse and poor nutrition are valid educational topics, but they do not relate as directly and frequently to childhood death as do injuries.

A 14-year-old boy has experienced a pronounced growth spurt over the last several months. While discussing this with his parents, the nurse educates what normal male growth patterns contain. Of the following, which are accurate statements to relay to the parents? Select all that apply. A) Most males will complete their growth spurt by age 16. B) It is not usual for their son to gain up to 30 kg in weight. C) With parathyroid hormone involvement, your son may be at risk for fractures. D) Expect the thorax to become broader and for the pelvis to remain narrow. E) Some children have stunted growth in their arms or legs.

B) It is not usual for their son to gain up to 30 kg in weight. D) Expect the thorax to become broader and for the pelvis to remain narrow. In males, they may continue to gain height until 18 to 20 years of age and gain from 7 to 30 kg of weight. Parathyroid hormone does not have roles that relate to the adolescent growth spurt. In males, the thorax becomes broader and the pelvis remains narrow. In girls, the opposite occurs. Growth in the arms, legs, hands, feet, and neck is followed by increases in the hip and chest months later.

A 3-year-old boy has developed croup following a winter cold. His care provider would recognize that which of the following microorganisms and treatments is most likely to be effective? A) Respiratory syncytial virus treated with intubation B) Parainfluenza virus treated with a mist tent and oxygen therapy C) Haemophilus influenza treated with appropriate antibiotics D) Staphylococcus aureus treated with bronchodilators and mist tent

B) Parainfluenza virus treated with a mist tent and oxygen therapy The majority of croup cases are caused by parainfluenza viruses, and common treatment modalities are humidified air or mist tents as well as supplementary oxygen. Respiratory syncytial virus accounts for some croup diagnoses, but intubation is not normally required. Haemophilus influenza is responsible for epiglottitis, while Staphylococcus aureus is not commonly responsible for croup.

A pneumonia that occurs 48 hours or more after admission to the hospital is considered A) community-acquired pneumonia. B) hospital-acquired pneumonia. C) viral pneumonia. D) immunocompromised pneumonia.

B) hospital-acquired pneumonia. Hospital-acquired pneumonia is defined as a lower respiratory tract infection that was not present or incubating on admission to the hospital. Usually, infections occurring 48 hours or more after admission are considered hospital acquired. Community-acquired pneumonia is diagnosed within 48 hours after admission. Most hospital-acquired pneumonia is bacterial.

A nurse is providing care for an older, previously healthy adult male who has been diagnosed today with pneumococcal pneumonia. Which of the following signs and symptoms is the nurse most likely to encounter? A) The man will be hypotensive and febrile and may manifest cognitive changes. B) The patient will have a cough producing clear sputum, and he will have faint breath sounds and fine crackles. C) The patient will have copious bloody sputum and diffuse chest pain and may lose his cough reflex. D) The patient will lack lung consolidation and will have little, if any, sputum production.

B) The patient will have a cough producing clear sputum, and he will have faint breath sounds and fine crackles. The typical onset of pneumococcal pneumonia involves production of clear sputum, along with faint breath sounds and fine crackles. The patient is less likely to be hypotensive, have copious bloody sputum, or have chest pain. A lack of lung consolidation or sputum production is more closely associated with atypical pneumonias.

A client with a newborn infant is also the caregiver for her 75-year-old mother, who lives with them and who has diabetes. The client requests pneumonia vaccinations for her entire household. Which vaccine is most likely to be effective for the baby? A) Since the baby's immune system is mature at birth, regular vaccine is appropriate. B) There is no effective vaccine for newborn infants. C) The 23-valent vaccine will be effective. D) No vaccine is necessary for the baby if the nursing mother is immunized

B) There is no effective vaccine for newborn infants. S. pneumoniae capsular polysaccharides would be especially appropriate for the client and her diabetic, elderly mother but is not effective in the immune system of anyone younger than 2 years old. Fortunately, a newer, 7-valent vaccine was designed to protect infants as young as 7 months. However, because their immune system is immature, the antibody response to most flu shots is poor or inconsistent in children younger than 2 years of age.

A patient is admitted for a relapse for sarcoidosis. Knowing this is usually caused by an inflammatory process, the nurse can anticipate administering A) a bronchodilator. B) a corticosteroid. C) aspirin. D) an albuterol inhaler.

B) a corticosteroid. Treatment is directed at interrupting the granulomatous inflammatory process that is characteristic of the disease and managing the associated complications. When treatment is indicated, corticosteroid drugs are used. Bronchodilators may be used if there is wheezing, but this is not a normal medication for this disease. Aspirin is a blood thinner. Albuterol is a short-term bronchodilator for acute asthma.

In the grocery store, a nurse overhears a teenage mother intentionally shaming and verbally reprimanding a child in public. The mother also grabbed the child's stuffed animal and tore the limbs off. From what the nurse remembers about abuse, this would be classified as a form of A) physical abuse. B) emotional abuse. C) sexual abuse. D) neglect.

B) emotional abuse. Emotional abuse or psychological maltreatment includes methods of verbal abuse, shaming, destruction of child's personal property, harming or killing child's pet, and bullying.

A health educator is performing a health promotion workshop with the staff of a large, urban homeless shelter, and a component of the teaching centers around tuberculosis. One of the staff members comments, "Anyone who's had contact with tuberculosis in the past can give it to any of the other residents of the shelter, even if they didn't get sick themselves." How could the educator best respond to this comment? A) "Many people do manage to fight off the infection, but you're right: they can still spread it by coughing or sneezing." B) "If someone has been previously exposed to tuberculosis, they are particularly infectious because they are often unaware of the disease." C) "Actually, people who have the latent form of the disease won't be sick and can't spread it either." D) "There isn't any real risk of them spreading it, but we would like to vaccinate everyone who's had any contact with it in the past."

C) "Actually, people who have the latent form of the disease won't be sick and can't spread it either." Contact with M. tuberculosis without the development of progressive primary tuberculosis results in a latent infection that is not communicable. Vaccination is not a common intervention in the United States.

New parents were just told by their physician that their son is two standard deviations above the mean. The parents later asked the nurse what that means. The nurse will explain by stating, A) "If your child is one standard deviation from the norm that translates to mean, he will be taller than 50% of his peers." B) "This is great news since it means you will have a larger baby than most." C) "Being two standard deviations above the mean translates into that your child will likely be taller than 95% of children in the population." D) "With the mean being average at 50%, two standard deviations means that your child will be at least 99.7% taller than his brother."

C) "Being two standard deviations above the mean translates into that your child will likely be taller than 95% of children in the population." The standard deviation determines how far a value varies or deviates from the mean. The points one standard deviation above or below the mean should include 68% of all values and two standard deviations 95% of all values. If a child's height is within one standard deviation of the mean, he is as tall as 68% of children in the population.

Which of the following statements made by parents of high schoolers would be a cause for the concern the child may be thinking about suicide? A) "My child seems to eat all the time. He tells me that all of his friends are eating a lot as well." B) "My child seems to go shopping at the mall every day after school with her friends. I think they hang out at the mall." C) "My child has never had problems in school until now. He is failing classes and getting in trouble." D) "My child used to talk to me about anything. Now she spends most of her time in her room texting friends."

C) "My child has never had problems in school until now. He is failing classes and getting in trouble." Risk factors for suicide in adolescents include substance abuse, personal or family history of depression, anxiety disorders, problems at school, problems communicating with parents, having a friend who committed suicide, and family ownership of a handgun.

When educating a student who lives in a crowded apartment and diagnosed with tuberculosis, the college school nurse will emphasize, A) "Once your fever goes away, you can stop taking the streptomycin injection." B) "If isoniazid makes you nauseous, we can substitute something milder." C) "To destroy this bacterium, you must strictly adhere to a long-term drug regimen." D) "You will have to wear an N95 mask while on campus at all times."

C) "To destroy this bacterium, you must strictly adhere to a long-term drug regimen." Success of chemotherapy for prophylaxis and treatment of tuberculosis depends on strict adherence to a lengthy drug regimen that includes isoniazid (INH), rifampin, ethambutol, pyrazinamide, and streptomycin (or some combination of these).

Due to rapid neural growth, a child can begin to control the bowel and bladder sphincters by what age? A) 12 months B) 18 months C) 2 years D) 4 years

C) 2 years The cephalocaudal proximodistal principle is followed as myelinization of the cortex, brain stem, and spinal cord is completed. The spinal cord is usually completely myelinated by 2 years of age. At that time, control of anal and urethral sphincters and motor skills of locomotion can be achieved.

The neonatal ICU nurse is aware that type II alveolar cells produce surfactant, and they usually develop at how many weeks of gestation? A) 17 to 18 weeks B) 19 to 20 weeks C) 24 to 28 weeks D) 34 to 38 weeks

C) 24 to 28 weeks Type II alveolar cells begin to develop at approximately 24 weeks. These cells produce surfactant, a substance capable of lowering the surface tension of the air-alveoli interface. By the 28th to 30th week, sufficient amounts of surfactant are available to prevent alveolar collapse when breathing begins.

Which of the following residents of a long-term care facility is most likely to be exhibiting the signs and symptoms of chronic obstructive pulmonary disease (COPD)? A) A 79-year-old lifetime smoker who is complaining of shortness of breath and pain on deep inspiration B) An 81-year-old smoker who has increased exercise intolerance, a fever, and increased white blood cells C) An 81-year-old male who has a productive cough and recurrent respiratory infections D) An 88-year-old female who experiences acute shortness of breath and airway constriction when exposed to tobacco smoke

C) An 81-year-old male who has a productive cough and recurrent respiratory infections Productive cough and recurrent respiratory infections are associated with COPD, while pain, fever, and increased white cells are not common signs and symptoms of COPD. Acute shortness of breath and bronchoconstriction are associated with asthma.

Which of the following clinical findings would be most closely associated with a client who has interstitial lung disease in comparison to chronic obstructive pulmonary disease (COPD)? A) Audible wheezing on expiration B) Reduced expiratory flow rates C) Decreased tidal volume D) Normal forced expiratory volume

C) Decreased tidal volume Because it takes less work to move air through the airways at an increased rate than it does to stretch a stiff lung to accommodate a larger tidal volume, interstitial lung disease is commonly associated with an increased respiratory rate but decreased tidal volume. Wheezing and decreased expiratory flow rate are more closely associated with COPD.

A physician is providing care for a child who has a diagnosis of cystic fibrosis (CF). Place the following events in the etiology of CF in ascending chronological order. Use all the options. A) Airway obstruction B) Recurrent pulmonary infections C) Impaired Cl- transport D) Decreased water content of mucociliary blanket E) Increased Na+ absorption

C) Impaired Cl- transport E) Increased Na+ absorption D) Decreased water content of mucociliary blanket A) Airway obstruction B) Recurrent pulmonary infections CF is associated with impaired Cl- transport and a consequent increase in Na+ absorption. These result in a lowered water content of the mucociliary blanket making it more viscid. These changes to the mucociliary blanket cause airway obstruction and, ultimately, pulmonary infections.

While assessing a premature infant born at 25 weeks' gestation, the neonatal intensive care unit (NICU) nurse would suspect which diagnosis when the infant displays poor muscle tone, apnea, and a new onset of somnolence? A) Hydrocephalus B) Airway obstruction C) Intraventricular hemorrhage D) Sepsis

C) Intraventricular hemorrhage Prematurity is a risk for IVH. Clinical manifestations are determined by the level of involvement. The most common symptoms are poor muscle tone, lethargy, apnea, decreased hematocrit, and somnolence.

A 66-year-old male presents to the emergency room accompanied by his wife who claims that he has been acting confused. The man is complaining of a sudden onset of severe weakness and malaise and has a dry cough and diarrhea. His temperature is 102.8°F, and his blood work indicates his sodium level at 126 mEq/L (normal 135 to 145 mEq/L). Based on this assessment, the nurse suspects the patient has A) bronchopneumonia. B) Mycoplasma pneumonia. C) Legionella pneumonia. D) pneumococcal pneumonia.

C) Legionella pneumonia. Confusion, dry cough, diarrhea, and hyponatremia are associated with Legionnaire disease and less so with bronchopneumonia, Mycoplasma pneumonia, or pneumococcal pneumonia.

A child with rhinosinusitis should be monitored for complications. Which of the following assessment findings would alert the nurse that a complication is developing? A) Purulent nasal discharge B) Temperature of 100.8°F C) Periorbital edema D) Complaints of headache

C) Periorbital edema Expected s/s of acute viral rhinosinusitis include facial pain, headache, purulent nasal discharge, decreased sense of smell, and fever. Complications can lead to intracranial and orbital wall problems. Facial swelling over the involved sinus, abnormal extraocular movements, protrusion of the eyeball, periorbital edema, or changes in mental status may indicate intracranial complications.

A short, nonsmoking 44-year-old male presents to the emergency room with left-sided chest pain and a cough. He states that the pain started abruptly and worsens with deep breathing and coughing. He denies recent injury. Assessment includes shallow respirations with a rate of 36, normal breath sounds, and no cyanosis. Which condition is most likely causing his symptoms? A) Myocardial infarction B) Spontaneous pneumothorax C) Pleuritis related to infection D) Obstructive atelectasis

C) Pleuritis related to infection Pleuritis, which frequently accompanies infections that cause cough, is unilateral, starts abruptly, and is worsened by coughing or deep breathing. The client's shallow, rapid breathing may be due to anxiety but also is a way of maintaining adequate air intake while avoiding deep breathing, which exacerbates the pain of pleuritis. His cough may be an indication of infection, especially as he is not a smoker. The pain of myocardial infarction is not worsened by deep breathing or coughing. Spontaneous pneumothorax would be very unlikely in a short, nonsmoking middle-aged man. Tachypnea might indicate obstructive atelectasis, but normal breath sounds and lack of cyanosis argue against it.

Which of the following assessment findings of a male infant 14 hours postpartum would be considered abnormal and would require further assessment and possible intervention? A) The baby's first stool appears to contain blood. B) The child is unable to breathe through his mouth. C) The baby's skin has a yellowish orange hue. D) The child's suck is weak when placed at his mother's breast.

C) The baby's skin has a yellowish orange hue. While not an uncommon event in early postnatal life, jaundice requires further assessment and possibly intervention. Meconium often contains blood, and young infants are exclusive nose breathers. A child's suck is frequently weak before it becomes established in the days to follow.

A nurse is performing a 5-minute Apgar score on a newborn female. Which of the following characteristics of the infant's current condition would not be reflected in the child's Apgar score? A) The baby's heart rate is 122 beats/minute. B) The infant displays a startle reflex when the crib is accidentally kicked. C) The child's temperature is 35.0°C (95°F) by axilla. D) The infant's skin is pink in color.

C) The child's temperature is 35.0°C (95°F) by axilla. While heart rate, color, and presence or absence of crying are all assessment criteria in the determination of an Apgar score, temperature is not a parameter that is measured.

In the early morning, an African American woman brings her 5-year-old son to the emergency room. The boy is wheezing, is short of breath, and has a dry cough. The mother states that he has always been very healthy. He went to bed with only a slight cold and a runny nose but woke her with his coughing shortly after 4 AM. His symptoms worsened so dramatically that she brought him to the hospital. The care team would most likely suspect that he has A) respiratory syncytial virus. B) influenza. C) asthma. D) pneumonia.

C) asthma. Although the child may have an infectious disease, his symptoms and the timing of them (both in terms of his age and the time of symptom onset) are classic for asthma. They are not as closely associated with RSV, influenza, or pneumonia

When looking at a granulocyte under a microscope, the anatomy student would describe it as a cell A) lacking granules. B) having a kidney-shaped nucleus. C) having no nuclei. D) shaped like a sphere with multilobar nuclei.

D) shaped like a sphere with multilobar nuclei. Granulocytes are spherical and have distinctive multilobar nuclei. They are all phagocytic cells that are identifiable because of their cytoplasmic granules. Answers A, B, and C are incorrect. Agranulocytes lack granular cytoplasm. Monocytes have a large amount of cytoplasm and a nucleus in the shape of a kidney. Erythrocytes lack a cell nucleus.

A geriatrician is following a number of clients on a subacute geriatric medical unit, some of whom require diagnostic blood work. Which of the following clients would be most likely to have an erythrocyte sedimentation rate (ESR) screening test ordered? An adult with A) a diagnosis of Alzheimer disease and depression. B) orthostatic hypotension and syncopal episodes. C) congestive heart failure. D) systemic lupus erythematosus.

D) systemic lupus erythematosus. Inflammatory diseases, such as systemic lupus erythematosus, are a common indicator for an ESR test. The other health problems noted are less commonly marked by inflammatory processes.

As part of a public health initiative, a nurse is teaching a group of older adults about ways to promote and maintain their health. Recognizing that the common cold is a frequent source of ailment, the nurse is addressing this health problem. Which of the following teaching points about the common cold is most accurate? A) "You shouldn't be taking antibiotics for a cold until your doctor has confirmed exactly which bug is causing your cold." B) "It's important to both cover your mouth when you cough or sneeze and encourage others to do so, since most colds are spread by inhaling the germs." C) "Scientists don't yet know exactly what virus causes the cold, and there is not likely to be a vaccine until this is known." D) "Use caution when choosing over-the-counter drugs for your cold; most people do best with rest and antifever medications."

D) "Use caution when choosing over-the-counter drugs for your cold; most people do best with rest and antifever medications." The efficacy of over-the-counter cold remedies is minimal, and all have a risk of unwanted side effects; rest and antipyretics are normally sufficient since cold viruses are normally self-limiting. No cold-causing virus will respond to antibiotics, and most colds are spread by the fingers. There is no one specific virus that causes the common cold, and numerous different viruses cause similar symptoms.

Which of the following infants most likely requires medical intervention? A) A 2-day-old baby boy who has caput succedaneum B) An infant 4 hours postpartum who has visible coning of his head following vaginal delivery C) A girl 3 days postpartum with noticeable unilateral cephalhematoma D) A male infant whose vertex delivery resulted in a brachial plexus injury

D) A male infant whose vertex delivery resulted in a brachial plexus injury While caput succedaneum, cephalhematoma, and head coning are all frequently able to resolve independently, a brachial plexus injury is likely to require treatment and rehabilitation.

A 10-year-old boy has a body mass index that places him in the 96th percentile for his age and gender. While educating the parents about obesity, the nurse should emphasize that his weight may predispose him to the development of A) scoliosis. B) respiratory infections. C) gastrointestinal disorders. D) type 2 diabetes

D) type 2 diabetes Adolescent obesity is associated with an increased risk of type 2 diabetes. He is less likely to face a heightened risk of scoliosis, respiratory infections, or GI disorders.

Which of the following situations would be most deserving of a pediatrician's attention? A) The mother of an infant 2 days postpartum notes that her baby has intermittent periods of hyperventilation followed by slow respirations or even brief periods of apnea. B) A volunteer in the nursery notes that one of the infants, aged 2 weeks, appears unable to breathe through his mouth, even when his nose is congested. C) A neonate is visibly flaring her nostrils on inspiration. D) A midwife notes that a newborn infant's chest is retracting on inspiration and that the child is grunting.

D) A midwife notes that a newborn infant's chest is retracting on inspiration and that the child is grunting. Retraction and grunting indicate a significant increase in the work of breathing that can be indicative of respiratory distress syndrome, a situation that would require medical intervention. Periods of hyperventilation interspersed with reduced breathing rates are common during the transition to postpartum ventilation, and infants are commonly unable to mouth breathe. Nostril flaring could be a sign of dyspnea, but it can also be a compensatory mechanism that the infant uses to increase oxygen intake; this situation would not be considered as serious as an infant who has chest retractions and grunting.

A 77-year-old lifetime smoker has been diagnosed with a tumor in his lung at the site of an old tubercle scarring site, located in a peripheral area of his bronchiolar tissue. What is this client's most likely diagnosis? A) Squamous cell carcinoma B) Small cell lung cancer C) Large cell carcinoma D) Adenocarcinoma

D) Adenocarcinoma Adenocarcinoma is associated with the periphery of the lungs, often at the site of scarring, and can occur in alveolar or bronchiolar tissue. Squamous cell carcinoma, small cell lung cancer, and large cell carcinoma are less commonly associated with these traits.

Which of the following statements best captures the etiology of the acute response phase of extrinsic (atopic) asthma? A) IgG production is heightened as a consequence of exposure to an allergen. B) Airway remodeling results in airflow limitations. C) Epithelial injury and edema occur along with changes in mucociliary function. D) Chemical mediators are released from presensitized mast cells.

D) Chemical mediators are released from presensitized mast cells. The acute response phase of extrinsic asthma is characterized by the release of chemical mediators from mast cells that have been sensitized. Epithelial injury and edema, as well as airway remodeling, are not associated with the acute phase, and IgE, not IgG, is primarily involved in asthma.

The exasperated parents of a 4-month-old infant with colic have asked their health care provider what they can do to alleviate their child's persistent crying. Based on their concerns, the nurse should educate/discuss with the parents which of the following? A) Encouraging them to walk away from the infant when they can no longer tolerate it B) Recommending them to reduce the amount of commercial formula and increase breast-feeding C) Discussing the use of prescribed antiflatulent medication that will help more than changing the formula D) Demonstrating how to use a soothing voice and slow rocking back and forth as a way to calm the infant

D) Demonstrating how to use a soothing voice and slow rocking back and forth as a way to calm the infant The lack of a single etiologic factor makes treatment of colic difficult. The incidence is similar with both breast-feeding and formula, and while antiflatulents are sometimes used, the problem is not always attributable to intestinal gas. Even though it is a common problem that does resolve with time, parents need support. Nonpharmacologic interventions include soothing voices, singing, swaddling, and slow rhythmic rocking

A woman has recently determined that she is pregnant, and her clinician believes that the conception occurred around 8 weeks prior. Since the embryo is in the third stage of embryonic development, which of the following events and processes in growth and development would be expected to be taking place? A) Transition from a morula to a blastocyst B) Ossification of the skeleton and acceleration of body length growth C) Rapid eye movement and early support of respiration D) Formation of upper limbs and opening of the eyes

D) Formation of upper limbs and opening of the eyes Limb formation and eye opening are associated with the third stage of the embryonic stage of development. The transition from a morula to a blastocyst occurs before the second week of gestation, while ossification of the skeleton and acceleration of body length growth do not take place until the early fetal period. Rapid eye movement and early pulmonary function emerge during the 26th through 29th weeks.

While rock climbing, a 22-year-old male has endured a severe head injury. Which of the following statements best captures expected clinical manifestations and treatments for his immediate condition? A) Oxygen therapy is likely to decrease his respiratory drive and produce an increase in PCO2. B) Cheyne-Stokes breathing is likely but will respond to bronchodilators. C) The client is unlikely to respond to supplementary oxygen therapy due to impaired diffusion. D) Hypoventilation may exist, resulting in increased PCO2 and hypoxemia that may require mechanical ventilation.

D) Hypoventilation may exist, resulting in increased PCO2 and hypoxemia that may require mechanical ventilation. Brain injuries and accompanying hypoventilation are often associated with increased PCO2 and by hypoxemia that responds to oxygen therapy. Persons with COPD are more vulnerable to diminished respiratory drive secondary to oxygen therapy, while Cheyne-Stokes breathing is not identified as a likely consequence of brain injury. Impaired alveolar diffusion is not an aspect of the client's injury.

Around 3 weeks after razing an old chicken house, a 71-year-old retired farmer has developed a fever, nausea, and vomiting. After ruling out more common health problems, his care provider eventually made a diagnosis of histoplasmosis. Which of the following processes is most likely taking place? A) Toxin production by Histoplasma capsulatum is triggering an immune response. B) Antibody production against the offending fungi is delayed by the patient's age and the virulence of the organism. C) Spore inhalation initiates an autoimmune response that produces the associated symptoms. D) Macrophages are able to remove the offending fungi from the bloodstream but can't destroy them

D) Macrophages are able to remove the offending fungi from the bloodstream but can't destroy them Disseminated histoplasmosis results from the inability of macrophages of the reticuloendothelial system to destroy the fungi. Fungi do not produce toxins, and antibody production and autoimmune responses are not involved in the pathophysiology of this fungal infection.

A patient diagnosed with low-risk chronic lymphocytic leukemia (CLL) has recently developed thrombocytopenia. One of the medications utilized to treat this would be A) dexamethasone, a corticosteroid. B) cisplatin, a chemotherapeutic. C) vincristine, a Vinca alkaloid. D) doxorubicin, a cytotoxic antibiotic.

A) dexamethasone, a corticosteroid. Complications of CLL include thrombocytopenia and may require treatment with corticosteroids or splenectomy. None of the other medications are used for treatment of CLL.

Which of the following statements best conveys a characteristic of red blood cells? A) They lack organelles and soluble enzymes. B) They contribute to the maintenance of blood pH. C) They help maintain the body's fluid balance. D) They are self-replicating.

B) They contribute to the maintenance of blood pH. Erythrocytes contribute to acid-base balance through the production of bicarbonate and hydrogen ions. They contain soluble enzymes but are neither self-replicating nor involved in fluid balance.

When talking about the various types of granulocytes, which granule contains heparin, an anticoagulant? A) Neutrophils B) Eosinophils C) Basophils D) Lymphocytes

C) Basophils The basophils have granules that contain heparin, an anticoagulant; histamine, a vasodilator; and other mediators of inflammation.

When talking about the lifespan of various blood components, the students should know that once a neutrophil moves into tissue, it lives approximately for how long? A) 12 hours B) 24 hours C) 2 days D) 4 days

D) 4 days After release from the marrow, the neutrophils spend only approximately 4 to 8 hours in the circulation before moving into the tissues. They survive in the tissues for approximately 4 to 5 days. They die in the tissues by discharging their phagocytic function.

A 22-year-old female who adheres to a vegan diet has been diagnosed with iron deficiency anemia. Which of the following components of her diagnostic blood work would be most likely to necessitate further investigation? A) Decreased mean corpuscular volume (MCV) B) Decreased hemoglobin and hematocrit C) Microcytic, hypochromic red cells D) Decreased erythropoietin levels

D) Decreased erythropoietin levels It would be unusual to note decreased levels of erythropoietin concurrent with a diagnosis of anemia. Decreased MCV, hematocrit, and hemoglobin are congruent with the diagnosis, as are microcytic, hypochromic erythrocytes.

A 30-year-old woman who has given birth 12 hours prior is displaying signs and symptoms of disseminated intravascular coagulation (DIC). The client's husband is confused as to why a disease of coagulation can result in bleeding. Which of the nurse's following statements best characterizes DIC? A) "So much clotting takes place that there are no available clotting components left, and bleeding ensues." B) "Massive clotting causes irritation, friction, and bleeding in the small blood vessels." C) "Excessive activation of clotting causes an overload of vital organs, resulting in bleeding." D) "The same hormones and bacteria that cause clotting also cause bleeding."

A) "So much clotting takes place that there are no available clotting components left, and bleeding ensues." DIC hemorrhage results from an insufficiency of clotting proteins after large-scale coagulation. It is not a result of physical irritation, organ overload, or bacteria and hormones.

Which of the following statements by a client of a cancer center who has a new diagnosis of non-Hodgkin lymphoma (NHL) demonstrates a sound understanding of the diagnosis and treatment of the health problem? A) "They confirmed my diagnosis with a lymph node biopsy, and I'll get radiation treatment soon because it's fairly early stage." B) "They took a sample of my lymph nodes, and I'll be having surgery soon that will hopefully cure my lymphoma." C) "My blood work came back positive for NHL, and I'm meeting with my oncologist to discuss chemotherapy soon." D) "Since the tests show NHL, I'm going to pursue my options for palliative care because I'm committed to dying with dignity."

A) "They confirmed my diagnosis with a lymph node biopsy, and I'll get radiation treatment soon because it's fairly early stage." Lymph node biopsy is often used to provide a diagnosis of NHL, and radiation is a common treatment, especially in early stages of the disease. Surgery is not a noted treatment modality, and while blood work may help with staging, it is not a common source of a confirming diagnosis. Treatment options do exist, and palliative care would be premature.

In which of the following individuals would a clinician most suspect multiple myeloma as a diagnosis? A) A 40-year-old man who has had three broken bones over the past 6 months and whose serum calcium and creatinine levels are elevated B) A 68-year-old former coal miner who has white cell levels exponentially higher than normal ranges C) An 81-year-old male resident of a long-term care home who has an uncommon bacterial pneumonia and who is unable to produce a fever D) A 70-year-old woman whose blood work reveals large numbers of immature granulocytes

A) A 40-year-old man who has had three broken bones over the past 6 months and whose serum calcium and creatinine levels are elevated The main sites involved in multiple myeloma are the bones and bone marrow. In addition to the abnormal proliferation of marrow plasma cells, there is proliferation and activation of osteoclasts that lead to bone resorption and destruction. This increased bone resorption predisposes the individual to pathologic fractures and hypercalcemia. Many patients also present with renal insufficiency. Leukostasis, susceptibility to infection, and disorders of granulocyte development are not hallmarks of multiple myeloma.

Which of the following patients would be at risk for developing nonthrombocytopenic purpura? Select all that apply. A) A child adopted from India and displaying malaise, lethargy, and petechiae all over the body B) A 73-year-old patient admitted with concussion that resulted from a fall C) A 55-year-old patient diagnosed with Cushing disease displaying bruises, weight gain with a buffalo hump, and moon face D) A 15-year-old insulin-dependent diabetic with hypoglycemia displaying irritability with headaches and tachycardia E) A pregnant mother experiencing headaches and proteinuria

A) A child adopted from India and displaying malaise, lethargy, and petechiae all over the body C) A 55-year-old patient diagnosed with Cushing disease displaying bruises, weight gain with a buffalo hump, and moon face Vascular disorders that cause bleeding include vitamin C deficiency (answer A), Cushing disease (answer C), senile purpura, and aging process. Answers B and D do not result in weak vessel walls initially. Pregnant mother experiencing headaches and proteinuria is associated with preeclampsia.

A 32-year-old woman presents at her neighborhood health clinic complaining of weakness and a feeling of abdominal fullness. She reports that 6 months earlier she noticed that she had difficulty in maintaining the high level of energy she has relied on during her aerobic workouts over the past few years. Because she felt that she was in overall good health, but knew that women often need additional iron, she added a multiple vitamin with iron and some meat and leafy greens to her diet. She followed her plan carefully but had no increase in energy. Upon examination, her spleen is noted to be enlarged. Which of the following is most likely to be the cause? A) Accelerated CML B) Stage A Hodgkin disease C) Infectious mononucleosis D) CLL

A) Accelerated CML Typically, CML follows a triphasic course: (1) a chronic phase of variable length, (2) a short accelerated phase, and (3) a terminal blast crisis phase. The onset of the chronic phase is usually slow with nonspecific symptoms. Anemia causes weakness, easy fatigability, and exertional dyspnea. The accelerated phase is characterized by enlargement of the spleen and progressive symptoms. Persons with Hodgkin disease are commonly designated as stage A if they lack constitutional symptoms and stage B if significant weight loss, fevers, pruritus, or night sweats are present. In cases of infectious mononucleosis, most persons seek medical attention for severe pharyngitis, which usually is most severe on days 5 through 7 and persists for a total of 7 to 14 days. CLL is mainly a disorder of older persons; fewer than 10% of those who develop the disease are younger than 50 years of age. Men are affected twice as often as women

Which of the following statements most accurately conveys an aspect of lymphatic system activity? A) B- and T-lymphocyte development begins in the bone marrow and ends in the peripheral lymphoid structures. B) B cells and macrophages are released from the bone marrow in their completed state. C) Stem cells in the lymph nodes initiate and regulate the process of white cell synthesis. D) Leukocytes bypass vascular circulation and are distributed instead by the lymphatic system.

A) B- and T-lymphocyte development begins in the bone marrow and ends in the peripheral lymphoid structures. While both precursor B and T lymphocytes begin their development in the bone marrow, they migrate to peripheral lymphoid structures to complete the differentiation process. Stem cells are not located in the lymph nodes, and circulation of white cells is not exclusive to the lymphatic circulatory system.

Following a bone marrow biopsy, which of the following assessments would indicate the patient is experiencing a complication as a result of this diagnostic procedure? A) BP 90/60, heart rate 132, excess bleeding, and hematoma noted at the insertion site B) BP 130/80, oxygen saturation 95%, and crackles heard on inspiration C) Respiratory rate 24 and complaining of pain at insertion site D) Heart rate regular 64 beats/minute and temperature 99.6°F orally

A) BP 90/60, heart rate 132, excess bleeding, and hematoma noted at the insertion site The major hazard of a bone marrow biopsy is the risk of hemorrhage. This is usually seen by a decrease in BP, tachycardia, edema from blood pooling in the tissues, etc. All of the other answers are s/s other than shock. Crackles in the lungs correlate with fluid/secretions in the lungs. A bone marrow aspiration is usually performed on the posterior iliac crest and should not cause an elevated respiratory rate. A low-grade fever is not considered a complication.

The emergency department is awaiting the arrival of a spinal cord-injured patient. Knowing the innervation of the diaphragm, a patient with which type of injury may be in need of immediate mechanical ventilation? Injury to the A) C4 area. B) C7 area. C) T1 area. D) T4 area.

A) C4 area. The diaphragm is the principal muscle of inspiration. It is innervated by the phrenic nerve roots, which arise from the cervical level of the spinal cord, mainly from C4 but also C3 and C5.

Two years after chemotherapy and radiation therapy for lung cancer, a 72-year-old patient notices that he seems to be extremely tired all the time. The physician suspects the patient may have developed aplastic anemia. The nurse assessing the patient will likely find which of the following clinical manifestations of aplastic anemia? Select all that apply. A) Complaints of weakness and fatigue B) Small spots of skin hemorrhages over the entire body C) Excess bleeding from gums and nose D) Spoon-shaped deformity of the fingernails E) Hemolysis from renal dialysis treatments.

A) Complaints of weakness and fatigue B) Small spots of skin hemorrhages over the entire body C) Excess bleeding from gums and nose The onset of aplastic anemia may be insidious or sudden. The initial presenting symptoms include weakness, fatigue, and pallor caused by the anemia. Petechiae and ecchymoses often occur on the skin, and bleeding from the nose, gums, vagina, or GI tract may occur due to decreased platelet levels. Spoon-shaped deformity of the fingernails is seen in iron deficiency anemia. Hemolysis and blood loss from renal dialysis treatments contribute to anemia associated with a deficiency of erythropoietin (which is normally produced in the kidneys).

A 71-year-old woman is dependent on oxygen therapy and bronchodilators due to her diagnosis of emphysema. Which of the following pathological processes occur as a result of her emphysema? Select all that apply. A) Decreased elastic recoil due to alveolar damage B) Decreased residual lung volume due to impaired alveolar ventilation C) Increased anatomical dead space due to reduced tidal volume D) Increased alveolar dead space due to incorrect intrapleural pressure

A) Decreased elastic recoil due to alveolar damage C) Increased anatomical dead space due to reduced tidal volume D) Increased alveolar dead space due to incorrect intrapleural pressure In lung pathology such as emphysema, large amounts of air are trapped at the end of a given breath, a situation that corresponds to increased residual volume and decreased vital capacity. Elastic recoil would tend to suffer, and both alveolar and anatomical dead space consequently increase.

A client with a diagnosis of atrophic gastritis and consequent pernicious anemia is receiving high oral doses of vitamin B12. Which of the following changes would be most likely expected by his care provider at the completion of his treatment? A) Decreased mean corpuscular volume B) Increased serum bilirubin C) Increased folic acid levels D) Decreased free heme levels

A) Decreased mean corpuscular volume Increased red cell size is associated with vitamin B12 deficiency, and MCV would decrease with treatment. Increased bilirubin and folic acid levels would not be associated with resolution of pernicious anemia, and heme is not normally present or measured in a free circulatory form.

Which of the following colony-stimulating factors (CSFs) is given to ESRD patients to help with their chronic anemia? A) Erythropoietin (EPO) B) Thrombopoietin (TPO) C) Neupogen D) Interleukin

A) Erythropoietin (EPO) The CSFs that act on committed progenitor cells include erythropoietin (EPO), which stimulates RBC production. TPO stimulates platelets. Neupogen is for white blood cell production. Interleukin is a cytokine utilized as an immune response agent.

Which of the following nursing interventions would be a priority when caring for a newborn who is receiving phototherapy for high bilirubin levels? Select all that apply. A) Frequent monitoring of temperature B) Keeping diapers dry and clean C) Maintaining oral intake to prevent dehydration D) Putting lotion on his skin frequently to prevent drying/cracking of skin E) Applying sunscreen to prevent ultraviolet radiation

A) Frequent monitoring of temperature C) Maintaining oral intake to prevent dehydration Effective treatment depends on the area of skin exposed and the infant's ability to metabolize and excrete bilirubin. Frequent monitoring of bilirubin levels, body temperature, and hydration is critical to the infant's care. Diapers are usually not worn when under phototherapy light (want to keep the skin exposed to the light). Applying lotion and sunscreen would result in possible burning of the infant's skin while under the light.

Which of the following glycoproteins is responsible for treating such diseases as bone marrow failure following chemotherapy and hematopoietic neoplasms such as leukemia? A) Growth factors and cytokines B) Neutrophils and eosinophils C) T lymphocytes and natural killer cells D) Natural killer cells and granulocytes

A) Growth factors and cytokines The identification and characterization of the various growth factors and cytokines have led to their use in treating a wide range of diseases like bone marrow failure, hematopoietic neoplasms, infectious diseases, and congenital and myeloproliferative disorders. Neutrophils are primarily responsible for maintaining normal host defenses against invading bacteria, cell debris, or foreign substances. Eosinophils increase in number during allergic reactions and parasitic infections. T lymphocytes are involved in cell-mediated immunity. Natural killer cells participate in innate or natural immunity, and their function is to destroy foreign cells. Granulocytes are phagocytic cells.

Which of the following assessments and laboratory findings would be most closely associated with acute leukemia? A) High blast cell counts and fever B) Decreased oxygen partial pressure and weight loss C) Increased serum potassium and sodium levels D) Increased blood urea nitrogen and bone pain

A) High blast cell counts and fever Acute leukemia is often marked by a fever as well as leukostasis. Changes in oxygen saturation, electrolytes, and BUN would be less closely associated with ALL and AML.

A 21-year-old male client has suffered a head injury during a crash on his motorcycle, and a deficit that assessments have revealed is an impaired swallowing mechanism. He has also developed aspiration pneumonia. Which of the following statements most accurately captures an aspect of his condition? A) His vocal folds are likely not performing their normal function. B) His epiglottis is covering his larynx. C) His vocal folds have been compromised. D) His tracheobronchial tree is intermittently obstructed.

A) His vocal folds are likely not performing their normal function. The vocal folds contribute to blocking of the airways during swallowing; compromise to this function is likely to allow food to enter the lungs. The epiglottis is performing its normal, protective role against aspiration when it covers the larynx, and the vocal folds contribute to sound enunciation, not swallowing or protection against aspiration. Tracheobronchial obstruction would not contribute to aspiration.

Following a course of measles, a 5-year-old girl developed scattered bruising over numerous body surfaces and was diagnosed with immune thrombocytopenic purpura (ITP). As part of her diagnostic workup, blood work was performed. Which of the following results is most likely to be considered unexpected by the health care team? A) Increased thrombopoietin levels B) Decreased platelet count C) Normal vitamin K levels D) Normal leukocyte levels

A) Increased thrombopoietin levels In ITP, thrombopoietin levels are not elevated. Platelet levels would be expected to be low, and vitamin K and leukocyte levels would be unlikely abnormal.

Misinterpreting her physician's instructions, a 69-year-old woman with a history of peripheral artery disease has been taking two 325 mg tablets of aspirin daily. How has this most likely affected her hemostatic status? A) Irreversible acetylation of platelet cyclooxygenase activity has occurred. B) The patient's prostaglandin (TXA2) levels are abnormally high. C) She is at risk of developing secondary immune thrombocytopenic purpura (ITP). D) The binding of an antibody to platelet factor IV produces immune complexes.

A) Irreversible acetylation of platelet cyclooxygenase activity has occurred. Aspirin can cause inhibition of platelet aggregation that lasts for the life of the platelet. High TXA2 levels would be associated with increased coagulability. ITP would not result from aspirin intake, and binding of an antibody to platelet factor IV is associated with heparin-induced thrombocytopenia.

When discussing the sequence of clot dissolution, the science instructor will talk about which item that begins the process? A) Plasminogen B) Dabigatran C) Platelets D) alpha2-plasmin inhibitor

A) Plasminogen As with clot formation, clot dissolution requires a sequence of steps controlled by activators and inhibitors. Plasminogen, the proenzyme for the fibrinolytic process, normally is present in the blood in its inactive form. It is converted to its active form, plasmin, by plasminogen activators formed in the vascular endothelium, liver, and kidneys. Dabigatran is an anticoagulant. Platelets actually help cells stick together or adhere.

A 71-year-old male patient with a history of myocardial infarction (MI) and peripheral vascular disease (PVD) has been advised by his family physician to begin taking 81 mg aspirin once daily. Which of the following statements best captures an aspect of the underlying rationale for the physician's suggestion? A) Platelet aggregation can be precluded through inhibition of prostaglandin production by aspirin. B) Aspirin helps to inhibit ADP action and minimizes platelet plug formation. C) Aspirin can reduce unwanted platelet adhesion by inhibiting TXA2 synthesis. D) Aspirin inhibits the conversion of fibrinogen into fibrin and consequent platelet plug formation.

A) Platelet aggregation can be precluded through inhibition of prostaglandin production by aspirin. Aspirin prevents platelet plug formation by inhibiting synthesis of prostaglandins that mediate clot formation. Aspirin does not influence ADP, TXA2 synthesis, or fibrinogen conversion.

A 29-year-old construction worker got a sliver under his fingernail 4 days ago. The affected finger is now reddened, painful, swollen, and warm to touch. Which of the following hematological processes is most likely occurring in the bone marrow in response to the infection? A) Proliferation of immature neutrophils B) High circulatory levels of myeloblasts C) Increased segmented neutrophil production D) Phagocytosis by myelocytes

A) Proliferation of immature neutrophils A part of the immune response is the proliferation of neutrophils in response to infectious organisms. Neutrophilia is an increase in immature neutrophils ("bands") most commonly seen in acute infections and tissue injuries that promote the accelerated release of neutrophils and their precursors into the circulation. Myeloblasts are not normally found in circulation, and segmented neutrophils would not yet have reached maturity (takes approximately 2 weeks to mature). Myelocytes do not perform phagocytosis.

A hospital laboratory technician is performing routine blood analysis as part of an inpatient's assessment and is examining the sample in a test tube following processing in a centrifuge and the addition of an anticoagulant. Which of the following observations would the technician most likely interpret as an anomaly? A) The bottom layer of blood in the tube accounts for around one third of the total volume. B) The middle layer of cells appears white to gray in color. C) The top layer of cells is too thin to visualize without microscopy. D) The yellowish fluid on the top of the sample appears to constitute around one half of the total volume

A) The bottom layer of blood in the tube accounts for around one third of the total volume. The body layer of cells in a centrifuged blood sample consists of erythrocytes and should constitute around 42% to 47% of the total blood volume. The middle cell layer of leukocytes should appear whitish gray, and the thrombocytes above that layer are frequently not discernable. The plasma layer should account for around 55% of the total blood volume.

Following a winter power outage, a client who had been using a home gasoline generator began to experience dizziness and headaches and was diagnosed with carbon monoxide poisoning. What is the goal of hyperbaric oxygen treatment for carbon monoxide poisoning? A) To increase the amount of oxygen carried in the dissolved state B) To increase the production of unbound hemoglobin C) To stimulate the release of oxygen at the capillaries D) To remove bound CO from hemoglobin

A) To increase the amount of oxygen carried in the dissolved state While increased alveolar PO2 improves the oxygen saturation of hemoglobin, carbon monoxide occupies the hemoglobin sites usually available for oxygen. With much of the blood composed, temporarily, of carboxyhemoglobin, it is necessary to rely upon alternate means to deliver oxygen to the tissues. Plasma's normally low carrying capacity for dissolved oxygen can be increased by administration of 100% oxygen in the high atmospheric pressure of a hyperbaric chamber

A newly diagnosed leukemia patient begins hemorrhaging from every orifice. The physician is concerned that the patient has developed disseminated intravascular coagulation (DIC). The nurse should anticipate which of the following orders to be prescribed for this patient? Select all that apply. A) Transfuse 2 units of platelets. B) Transfuse fresh frozen plasma. C) Give aspirin twice per day. D) Administer IV Toradol stat. E) Place in reverse isolation.

A) Transfuse 2 units of platelets. B) Transfuse fresh frozen plasma. The treatment of DIC is directed toward managing the primary disease, replacing clotting components, and preventing further activation of clotting mechanisms. Transfusions of FFP, platelets, or fibrinogen-containing cryoprecipitate may correct the clotting factor deficiency. ASA would make the bleeding worse. Toradol is an NSAID and should be avoided in patients with a bleeding problem. Reverse isolation is implemented for patients with pancytopenia.

A patient with rheumatoid arthritis has been diagnosed with a secondary immune-associated neutropenia called Felty syndrome. The nurse has had to research this and found that she should be assessing this patient for which of the following manifestations of Felty syndrome. Select all that apply. A) Upper left quadrant pain on palpation B) An area of diminished breath sounds related to pneumonia C) Intermittent pain that radiates from the flank to the groin D) Swelling and pain in all joints when put through full range of motion E) Headache that worsens when exposed to bright lights

A) Upper left quadrant pain on palpation B) An area of diminished breath sounds related to pneumonia Felty syndrome, a variant of rheumatoid arthritis (RA), is a triad of splenomegaly (upper left quad pain), recurrent pulmonary infections (diminished breath sounds), and neutropenia. Intermittent pain that radiates from the flank to the groin is usually associated with kidney stone pain. Swelling and pain in all joints when put through full range of motion are also associated with rheumatoid arthritis.

A 44-year-old woman has developed calf pain during a transatlantic flight. She is extremely short of breath upon arrival at her destination. She was subsequently diagnosed with a pulmonary embolism (PE) that resolved with anticoagulant therapy. Which of these statements best characterizes the underlying problem of her PE? A) Ventilation was occurring, but perfusion was inadequate causing shortness of breath. B) The combination of normal perfusion but compromised ventilation caused hypoxia. C) She developed a transient anatomic shunt resulting in impaired oxygenation. D) Impaired gas diffusion across alveolar membranes resulted in dyspnea and hypoxia.

A) Ventilation was occurring, but perfusion was inadequate causing shortness of breath. Impaired blood flow to a portion of the lung, such as with a PE, is associated with ventilation without perfusion, rather than perfusion without ventilation. The situation is not related to an anatomic shunt or impaired diffusion across alveolar membranes.

A 60-year-old male hospital patient with a diagnosis of chronic obstructive pulmonary disease (COPD) is undergoing lung function tests to gauge the progression of his disease. Which of the following aspects of the lung volumes will the respiratory therapist be most justified in using to guide interpretation of the test results? A) Vital capacity will equal the patient's combined inspiratory reserve, expiratory reserve, and tidal volume. B) Vital capacity will equal the total lung capacity. C) Resting tidal volume will exceed that of tidal volume during activity. D) Expiratory reserve will equal residual lung volume.

A) Vital capacity will equal the patient's combined inspiratory reserve, expiratory reserve, and tidal volume. Vital capacity is determined by combining inspiratory reserve, expiratory reserve, and tidal volume. Total lung capacity always exceeds vital capacity, given that it is not possible to completely empty the lungs. Tidal volume becomes wider during exercise, and expiratory reserve is neither equal to nor synonymous with residual volume.

During science class, a student asks, "What's the difference between plasma and serum in the blood?" The nurse responds that the primary difference between plasma and serum is that plasma contains A) fibrinogen. B) heparin. C) white blood cells. D) hydrogen ions.

A) fibrinogen. The major difference between plasma and serum is the presence of fibrinogen in the plasma from an anticoagulated centrifuged whole-blood specimen. When blood is removed from the body for testing, it clots within 30 to 60 minutes. The clot contains the blood's cellular components enmeshed in an insoluble fibrin network (formed by the polymerization of soluble plasma protein fibrinogen). The remaining fluid portion is the yellow liquid serum. This serum no longer contains fibrinogen because the fibrinogen originally present in uncoagulated blood was used up in the formation of the blood clot. Heparin, WBC, and hydrogen ions are incorrect.

Following an injury resulting in a small cut from a knife, the first cells to go to the area of the cut would be the A) neutrophils. B) erythrocytes. C) albumin. D) basophi

A) neutrophils. Neutrophils are very mobile and are the first cells to go to an area of tissue damage. Erythrocytes (RBCs) contain an O2-carrying protein that functions in the transport of oxygen. Albumin, a plasma protein, does not pass through the pores in the capillary wall to enter the interstitial fluid and therefore contributes to the plasma osmotic pressure and maintenance of blood volume. Basophil granules contain heparin, an anticoagulant; histamine, a vasodilator; and other mediators of inflammation.

A student makes the statement to a colleague, "Blood plasma is essentially just a carrier for the formed elements like red blood cells and white blood cells." What would be the most accurate response to this statement? A) "That's not really true. Plasma is crucial in the immune and inflammatory responses." B) "Actually, plasma plays a significant role in nutrient and waste transport." C) "Not really. Plasma also contributes to the processes of protein synthesis and hematopoiesis." D) "Actually, plasma is integral to the proper function of the liver and maintenance of acid-base balance."

B) "Actually, plasma plays a significant role in nutrient and waste transport." Plasma proteins are integral to transport. They are not noted to contribute to the inflammatory response, hematopoiesis, protein synthesis, or liver function.

The family members of an elderly patient are wondering why his "blood counts" are not rising after his last GI bleed. They state, "He has always bounced back after one of these episodes, but this time it isn't happening. Do you know why?" The nurse will respond based on which of the following pathophysiological principles? A) "Everything slows down when you get older. You just have to wait and see what happens." B) "Due to stress, the red blood cells of older adults are not replaced as promptly as younger people." C) "The doctor may start looking for another cause of his anemia, maybe cancer of the bone." D) "Don't worry about it. We can always give him more blood."

B) "Due to stress, the red blood cells of older adults are not replaced as promptly as younger people." In older adults, the number of progenitor cells declines. During a stress situation such as bleeding, the red blood cells of older adults are not replaced as promptly as those of their younger counterparts. Given the scenario, the patient is obviously bleeding from the GI tract. There is no reason to suspect the patient has bone cancer. Answer choice D is a nontherapeutic communication technique. The nurse is trying to pacify the family and not really addressing their concern.

A surgeon is explaining to the parents of a 6-year-old boy the rationale for the suggestion of removing the boy's spleen. Which of the following teaching points would be most accurate? A) "Ferritin is the activated and usable form of iron that your red blood cells can use to transport oxygen." B) "Ferritin is a stored form of iron that indirectly shows me whether you would benefit from iron pills." C) "Ferritin is a protein-iron complex that allows your red blood cells to make use of the iron that you consume in your diet." D) "Ferritin is the form of iron that is transported in your blood plasma to red blood cells that need it."

B) "Ferritin is a stored form of iron that indirectly shows me whether you would benefit from iron pills." Ferritin is the protein-iron complex that is stored in tissues, especially the liver, and the serum levels can be used as an indicator of the need for iron supplements. It is not accurately characterized as an activated form of iron, nor does it mediate the actual use of iron by erythrocytes. Transferrin, not ferritin, is transported in plasma, and it is associated with iron storage.

A 70-year-old woman has received a diagnosis of chronic myelogenous leukemia (CML) after a clinical investigation sparked by the presence of leukocytosis in her routine blood work. What clinical course should her care provider tell her to expect? A) "You can expect your blood results, fatigue, and susceptibility to infection to gradually worsen over a few years." B) "You could remain the chronic stage of CML for several years before it accelerates and culminates in a crisis." C) "It's likely that this will give you chronic fatigue and malaise for the rest of your life, but that will probably be the extent of your symptoms." D) "Unfortunately, your leukemia will likely enter a crisis mode within a few weeks if we don't treat it immediately."

B) "You could remain the chronic stage of CML for several years before it accelerates and culminates in a crisis." The clinical course of CML is commonly divided into three phases: (1) a chronic phase of variable length, (2) a short accelerated phase, and (3) a terminal blast crisis phase. Persons in the early chronic phase of CML generally are asymptomatic, but without effective treatment, most will enter the accelerated phase within 4 years.

Which of the following patients has an absolute neutrophil count that is critically low and that the standard of care would recommend they be placed on neutropenic precautions? A) A patient on long-term steroids for rheumatoid arthritis with WBC of 7000 B) A 37-year-old patient with leukemia being treated with chemotherapy with ANC of 400 C) A 65-year-old prostate cancer patient receiving radiation therapy with neutrophil count of 2000 D) A 75-year-old renal failure patient receiving Epogen for anemia with hemoglobin level of 9.7

B) A 37-year-old patient with leukemia being treated with chemotherapy with ANC of 400 Neutrophils constitute the majority of blood leukocytes and play a critical role in host defense mechanisms against infections. The ANC is supposed to be 1000/µL, and if the ANC is less than 500 cells/mm3 , the person is generally put on neutropenic precautions in the hospital to protect him or her from the environment. A patient on long-term steroids for rheumatoid arthritis with WBC of 7000 has a normal WBC count; a prostate cancer patient receiving radiation therapy with neutrophil count of 2000 has a normal neutrophil count; a renal failure patient receiving Epogen for anemia with hemoglobin level of 9.7 is associated with RBCs and not neutrophils.

A nurse is providing care for several patients on an acute medical unit of a hospital. Which of the following patients would be most likely to benefit from hematopoietic growth factors? A) A 51-year-old female patient with liver failure secondary to hepatitis B) A 61-year-old female patient with end-stage renal cancer C) A 55-year-old obese male patient with peripheral neuropathy secondary to diabetes D) A 44-year-old man with a newly diagnosed brain tumor

B) A 61-year-old female patient with end-stage renal cancer Hematopoietic growth factors are commonly used in cases of kidney disease and cancer. Erythropoietin (EPO) is a colony-stimulating factor that stimulates red blood cell production. As a review, erythropoietin is an essential hormone for red cell production. Without it, definitive erythropoiesis does not take place. Under hypoxic conditions, the kidney will produce and secrete erythropoietin to increase the production of red blood cells. With cancer of the kidney, the kidney is unable to perform its normal function. They are not as clearly indicated in cases of liver disease, diabetes, or brain tumors.

While being on subcutaneous heparin injections for deep vein thrombosis during her latter pregnancy, a patient begins to experience major side effects. Her OB-GYN physician has called in a specialist who thinks that the patient is experiencing heparin-induced thrombocytopenia. The nurse should anticipate which of the following orders? A) Decrease the dose of heparin from 5000 units b.i.d to 3000 units b.i.d B) Immediately discontinue the heparin therapy C) Switch to Coumadin 2.5 mg once/day D) Infuse FFP stat

B) Immediately discontinue the heparin therapy The treatment of HIT requires the immediate discontinuation of heparin therapy and the use of alternative anticoagulants to prevent thrombosis recurrence. Decreasing the dose will not stop HIT. Coumadin is contraindicated in pregnancy. FFP is not called for in this situation

A 13-year-old African American boy comes to the ER complaining of fatigue and a rapid heartbeat. In conversation with the father, it becomes apparent to you that the boy has grown 2 inches in the previous 5 months. What is the first problem the health care team would attempt to rule out? A) Sickle cell anemia B) Iron deficiency anemia C) Thalassemia D) Aplastic anemia

B) Iron deficiency anemia Although each of the above answers is associated with fatigue and rapid heartbeat, male adolescents are particularly susceptible to iron deficiency anemia. They have high iron requirements because of growth spurts and dietary deficiencies.

A male, lifetime smoker has died because of chronic obstructive pulmonary disease. Which of the following phenomena regarding his alveoli would his care team expect in the weeks prior to his death? A) Proliferation of natural killer (NK) cells in the alveolar lumen B) Large numbers of alveolar macrophages in septal connective tissue C) The presence of tubercles in the interalveolar spaces D) Compensatory regeneration of type I alveolar cells

B) Large numbers of alveolar macrophages in septal connective tissue Smokers often retain large numbers of carbon-filled macrophages in their septal connective tissue. NK cell proliferation is not a noted phenomenon in the alveoli, and tubercles are associated specifically with tuberculosis infection. Type I alveoli are incapable of regeneration.

A 30-year-old male's blood work and biopsies indicate that he has proliferating osteoclasts that are producing large amounts of IgG. What is the man's most likely diagnosis? A) Acute myelogenous leukemia B) Multiple myeloma C) Acute lymphocytic leukemia D) Hodgkin lymphoma

B) Multiple myeloma One of the characteristics resulting from the proliferating osteoclasts in multiple myeloma is the unregulated production of a monoclonal antibody referred to as the M protein. In most cases, the M protein is either IgG or IgA. This phenomenon is not present in cases of CML, ALL, or Hodgkin lymphoma.

Which of the following statements best captures an aspect of the process of hematopoiesis? A) Colony-stimulating factors (CSFs) produce cytokines that activate progenitor cells. B) Progenitor cells differentiate into precursor cells. C) Various subtypes of pluripotent stem cells eventually differentiate into the cellular components of blood. D) Self-replicating precursor cells differentiate into specific CSFs.

B) Progenitor cells differentiate into precursor cells. During hematopoiesis, progenitor cells differentiate into precursor cells, which in turn differentiate into the cellular components of blood. Cytokine stimulation precedes and regulates the process, and there are no different subtypes of pluripotent stem cells. Precursor cells are not self-replicating.

Reviewing pathology for an exam on pulmonary vasculature, the nursing student states that blood enters the right side of the heart via the vena cavae, then to the right atrium, right ventricle, and then which vessel carries the deoxygenated blood into the pulmonary system? A) Pulmonary capillaries B) Pulmonary artery C) Pulmonary vein D) Ductus arteriosus

B) Pulmonary artery Deoxygenated blood leaves the right heart through the pulmonary artery. Return of oxygenated blood to the heart occurs by way of the pulmonary vein, which empties into the left atrium.

A 66-year-old female patient has presented to the emergency department because of several months of intermittently bloody stool that has recently become worse. The woman has since been diagnosed with a gastrointestinal bleed secondary to overuse of nonsteroidal anti-inflammatory drugs that she takes for her arthritis. The health care team would realize that which of the following situations is most likely to occur? A) The woman has depleted blood volume due to her ongoing blood loss. B) She will have iron deficiency anemia due to depletion of iron stores. C) The patient will be at risk for cardiovascular collapse or shock. D) She will have delayed reticulocyte release.

B) She will have iron deficiency anemia due to depletion of iron stores. Ongoing blood loss is associated with iron deficiency anemia due to the depletion of iron stores. She is unlikely to have a depleted blood volume or be at risk for shock, situations more commonly associated with traumatic, sudden blood loss. There would not likely be a delay in the release of reticulocytes.

A hospital client is receiving intravenous infusion of heparin for treatment of a pulmonary embolus. Which of the following phenomena is most likely to occur, resulting in the drug's therapeutic effect? A) Inhibition of vitamin K synthesis in the liver B) Suppression of fibrin formation C) Deactivation of the intrinsic clotting pathway D) Inhibition of ADP-induced platelet aggregation

B) Suppression of fibrin formation Ultimately, heparin inhibits the clotting factors that mediate the formation of fibrin. It does not inhibit vitamin K synthesis, nor does it deactivate the intrinsic clotting pathway in particular. Heparin does not act on platelet plug formation.

A patient asks the health care provider why his lower legs look purple. The health care provider will base her response on which pathophysiological principle? A) Too much trauma breaks capillaries, and they bleed into the tissue. B) The bruising around the ankles is due to the fact that it is a dependent area where the capillary pressure is higher. C) There is a problem with his plasminogen levels. D) Morbid obesity causes veins to enlarge and bleed into tissues due to stress the abdomen is placing on the vascular system.

B) The bruising around the ankles is due to the fact that it is a dependent area where the capillary pressure is higher. Cutaneous bleeding is seen as pinpoint hemorrhages and purple areas of bruising in dependent areas where the capillary pressure is higher. There is no indication that the patient has experienced trauma to the area or is morbidly obese. Plasminogen helps with clot dissolution.

A hospital laboratory technologist is analyzing the complete blood count (CBC) of a patient. Which of the following statements best reflects an aspect of the platelets that would constitute part of the CBC? A) Platelets originate with granulocyte colony-forming units (CFU). B) The half-life of a platelet is typically around 8 to 12 days. C) The alpha-granules of platelets contribute primarily to vasoconstriction. D) New platelets are released from the bone marrow into circulation.

B) The half-life of a platelet is typically around 8 to 12 days. Platelets' half-life is typically around 8 to 12 days. They originate from megakaryocytes, and -granules facilitate vasoconstriction. New platelets are released from the spleen into circulation.

A 60-year-old woman is suspected of having non-Hodgkin lymphoma (NHL). Which of the following aspects of her condition would help to rule out Hodgkin lymphoma? A) Her neoplasm originates in secondary lymphoid structures. B) The lymph nodes involved are located in a large number of locations in the lymphatic system. C) The presence of Reed-Sternberg cells has been confirmed. D) The woman complains of recent debilitating fatigue.

B) The lymph nodes involved are located in a large number of locations in the lymphatic system. While NHLs tend to be multicentric, Hodgkin lymphoma tends to involve a single node or group of nodes. The presence of Reed-Sternberg cells would indicate Hodgkin lymphoma, and both NHL and Hodgkin lymphoma involve secondary lymphoid structures and would cause fatigue.

Which of the following situations is most likely to result in an increased binding affinity of hemoglobin for oxygen? A) A client is in respiratory acidosis, with a low pH. B) Three of four binding sites on a client's hemoglobin molecule are occupied by oxygen. C) A client's body temperature is elevated as a result of an infectious process. D) An increase in 2,3-diphosphoglycerate enhances the loading of oxygen.

B) Three of four binding sites on a client's hemoglobin molecule are occupied by oxygen. As each binding site on a hemoglobin molecule is occupied, the affinity of the remaining sites for oxygen binding is increased. Increased affinity is associated with alkalosis, not acidosis, and fever causes reduced affinity. Exercise increases the unloading of oxygen, a situation characterized by low affinity.

A 14-year-old boy has been diagnosed with infectious mononucleosis. Which of the following pathophysiological phenomena is most responsible for his symptoms? A) The Epstein-Barr virus (EBV) is lysing many of the boy's neutrophils. B) Viruses are killing some of his B cells and becoming incorporated into the genome of others. C) The EBV inhibits the maturation of white cells within his peripheral lymph nodes. D) The virus responsible for mononucleosis inhibits the maturation of myeloblasts into promyelocytes

B) Viruses are killing some of his B cells and becoming incorporated into the genome of others. B lymphocytes all of which have receptors for the EBV that causes mononucleosis. Infection of the B cells may take one of two forms—it may kill the infected B cell, or it may become incorporated into its genome. Lysis of neutrophils or inhibition of white cell maturation or differentiation is not a central component of mononucleosis etiology

A 44-year-old female patient presents to the emergency department with abnormal bleeding and abdominal pain that is later attributed to gallbladder disease. Which of the following diagnoses would the medical team be most justified in suspecting as a cause of the patient's bleeding? A) Excess calcium B) Vitamin K deficiency C) Hemophilia B D) Idiopathic immune thrombocytopenic purpura (ITP)

B) Vitamin K deficiency Factors VII, IX, and X and prothrombin require the presence of vitamin K for normal activity. Vitamin K deficiency may result from impaired fat absorption caused by liver or gallbladder disease. Calcium, factors X and V, and platelet phospholipids combine to form prothrombin activator, which then converts prothrombin to thrombin. Excess calcium could result in increased formation of blood clots. Hemophilia B is a hereditary disorder. Half of the cases of ITP occur as an acute disorder in children; ITP in adults is a chronic disorder with insidious onset.

A nurse in an acute medical unit is providing care for a number of patients with a variety of diagnoses. Which of the following patients most likely exhibits risk factors for impaired coughing? A patient with A) an injury to her cerebellum. B) a nasogastric (NG) tube attached to suction. C) a diagnosis of viral pneumonia. D) diagnosis of diabetes mellitus and morbid obese.

B) a nasogastric (NG) tube attached to suction. An NG tube can inhibit the closing of the upper airways that is required for normal coughing. Pneumonia, obesity, diabetes, and injury to the cerebellum are unlikely to affect the ability to cough.

While working in the newborn ICU, the nurses receive a call that an infant, gestational age of 23 weeks, is being air flighted to the level 3 trauma nursery. The priority intervention for this infant would be A) insertion of an umbilical line for fluids. B) intubation and mechanical ventilation. C) insertion of a feeding tube. D) insertion of an intraventricular catheter.

B) intubation and mechanical ventilation. The type II alveolar cells that produce surfactant do not begin to mature until 26th to 27th week of gestation; consequently, many premature infants have difficulty in producing sufficient amounts of surfactant. This can lead to alveolar collapse and severe respiratory distress. The only answer (B) to facilitate respiratory is mechanical ventilation. IV fluids and nutrition are important but not a priority of airway/breathing problems. There is no indication that the infant has increased ICP and would need an intraventricular catheter.

A patient has visited his physician because he found an enlarged lymph node along the mediastinal border. When questioned, the physician may be alerted to a possible diagnosis of Hodgkin lymphoma (HL) if the patient also displays: Select all that apply. A) yeast infection in the mouth. B) night sweats. C) unexplained pruritus. D) joint swelling. E) sore throat with pustules on tonsils.

B) night sweats. C) unexplained pruritus. Most people with HL present with painless enlargement of a single node or group of nodes. The initial lymph node involvement typically is above the level of the diaphragm. Mediastinal masses are frequent and discovered on routine chest x-ray. There may be complaints of chest discomfort with cough or dyspnea. Additional symptoms include fevers, chills, night sweats, and weight loss. Pruritus and intermittent fevers associated with night sweats are classic symptoms of HL. Yeast infections and joint swelling are not associated with HL. Sore throat with pustules on tonsils is associated with strep throat.

The nurse is hearing diminished breath sounds and a "grating" sound during respirations. This is consistent with excess collection of fluid in the pleural cavity. The medical term for this is A) pleurisy. B) pleural effusion. C) pneumothorax. D) poor lung compliance.

B) pleural effusion. Pleural effusion is used to describe an abnormal collection of fluid or exudates in the pleural cavity. Pleurisy is an inflammation in the pleural space, and pneumothorax is an abnormal collection of air in the pleural space.

A 44-year-old Caucasian woman is being treated in an airport infirmary after she developed a painful, swollen leg during a transatlantic flight in economy class. The woman is suspected of having deep vein thrombosis (DVT) and is questioning the paramedics about why this might be the case, given that she has twice previously had similar experiences. Which of the following teaching points by the airport medical staff would be most appropriate? A) "A lot of Caucasian people have a genetic mutation that causes platelets to stick to their blood vessel walls." B) "There is a genetic disorder that causes many Caucasians to form more clots in their blood vessels." C) "A lot of Caucasians have an inherited inability to dissolve clots that form in their bodies." D) "Your doctor might be able to tell you if you've inherited a predisposition to bleeding in your veins."

C) "A lot of Caucasians have an inherited inability to dissolve clots that form in their bodies." All of the listed responses refer to the Leiden mutation, which is best characterized as an inhibition of normal clot dissolution due to factor V defects. It does not involve platelet aggregation or adhesion or excess bleeding. It is better characterized as decreased clot dissolution rather than increased clot formation.

Two nursing students are attempting to differentiate between the presentations of immune thrombocytopenic purpura (ITP) and thrombotic thrombocytopenic purpura (TTP). Which of the students' following statements best captures an aspect of the two health problems? A) "Both diseases can result from inadequate production of thrombopoietin by megakaryocytes." B) "ITP can be either inherited or acquired, and if it's acquired, it involves an enzyme deficiency." C) "Both of them involve low platelet counts, but in TTP, there can be more, not less, hemostasis. D) "TTP can be treated with plasmapheresis, but ITP is best addressed with transfusion of fresh frozen plasma."

C) "Both of them involve low platelet counts, but in TTP, there can be more, not less, hemostasis. TTP is marked by sudden and severe thrombotic involvement. Neither disease has an etiology of low thrombopoietin production, and TTP, not ITP, is rooted in an enzyme deficiency. ITP is normally treated with corticosteroids and/or immunoglobulins.

Which of the following teaching points would be most appropriate with a client who has a recent diagnosis of von Willebrand disease? A) "It's important that you avoid trauma." B) "Your disease affects your platelet function rather than clot formation." C) "Make sure that you avoid taking aspirin." D) "Clotting factor VIII can help your body compensate for the difficulty in clotting."

C) "Make sure that you avoid taking aspirin." No treatment other than the avoidance of aspirin is normally needed in the case of von Willebrand disease. Avoiding trauma and factor VIII therapy apply to hemophilia. Von Willebrand disease involves both the platelet and coagulation systems.

A surgeon is explaining to the parents of a 6-year-old boy the rationale for the suggestion of removing the boy's spleen. Which of the following teaching points would be most accurate? A) "We believe that your son's spleen is causing the destruction of many of his blood platelets, putting him at a bleeding risk." B) "Your son's spleen is inappropriately filtering out the platelets from his blood and keeping them from normal circulation." C) "Your son's spleen is holding on to too many of his platelets, so they're not available for clotting." D) "We think that his spleen is inhibiting the production of platelets by his bone marrow."

C) "Your son's spleen is holding on to too many of his platelets, so they're not available for clotting." A cause of thrombocytopenia is excessive sequestering of platelets by the spleen, necessitating splenectomy. The spleen would not be involved in destroying platelets, filtering them out from existing circulation, or inhibiting their production.

A couple who is expecting their first child has been advised by friends to consider harvesting umbilical cord blood in order to have a future source of stem cells. The couple has approached their caregiver with this request and is seeking clarification of exactly why stem cells are valuable and what they might expect to gain from harvesting it. How can their caregiver best respond to the couple's enquiry? Stem cells can A) "help correct autoimmune diseases and some congenital defects." B) "be used to regenerate damaged organs should the need ever arise." C) "be used as source of reserve cells for the entire blood production system." D) "help treat some cancers and anemias, but they must come from your child himself or herself."

C) "be used as source of reserve cells for the entire blood production system." The proliferative potential and self-renewal of stem cells make them a compensatory mechanism and reserve source for the entire hematopoietic system. While they could be of possible use in certain autoimmune conditions or in cases of organ failure, these statements do not capture their essence. Stem cells can also come from a histocompatible donor.

Which of the following neurological patients is most likely to have abnormalities in breathing regulation? A) A 23-year-old male who has an injury to his frontal lobe following a sports injury B) A 45-year-old female with a spinal cord injury at C7 following a motor vehicle accident C) A 34-year-old male with damage to his upper and lower pons following a blow to the back of the head D) A 66-year-old male with temporal lobe infarcts secondary to a stroke

C) A 34-year-old male with damage to his upper and lower pons following a blow to the back of the head The respiratory center is located in the pons. Damage to the temporal lobe, frontal lobe, or spinal cord at C7 is less likely to affect respiration.

Your ESRD patient is receiving 2 units of packed red blood cells for anemia (Hgb of 8.2). Twenty minutes into the first transfusion, the nurse observes the patient has a flushed face, hives over upper body trunk, and is complaining of pain in lower back. His vital signs include pulse rate of 110 and BP drop to 95/56. What is the nurse's priority action? A) Slow the rate of the blood infusion to 50 mL/hour. B) Document the assessment as the only action. C) Discontinue the transfusion and begin an infusion of normal saline. D) Recheck the type of blood infusing with the chart documentation of patient's blood type.

C) Discontinue the transfusion and begin an infusion of normal saline. An immediate hemolytic reaction usually is caused by ABO incompatibility. The signs include flushing of the face, urticaria (hives), headache, pain in the lumbar area, chills, fever, chest pain, tachycardia, hypotension, and dyspnea. If any of these actions occur, the transfusion should be stopped immediately. Access to a vein should be maintained because it may be necessary to infuse IV solutions to ensure diuresis. Slowing the rate of the blood infusion will not correct this hemolytic reaction and will only worsen the patient's condition. Of course, documentation after the above interventions are performed is vital. Rechecking the blood type infusing will not stop the hemolytic reaction. After corrective actions/interventions are taken, the blood bag is returned to the blood bank for further testing

A 62-year-old female with a diagnosis of acute and chronic renal failure secondary to diabetes mellitus is receiving her weekly injection of epoetin, a supplementary form of erythropoietin. Which of the following statements best captures the necessity of this medication? A) Erythropoietin is needed in order for stem cells to proliferate into committed erythroid precursors. B) Erythropoietin is necessary for the accurate sensation of hypoxia that stimulates erythropoiesis. C) Erythropoietin causes the erythrocyte colony-forming units to proliferate and mature. D) Erythropoietin facilitates the extrusion of the reticulocyte nucleus and the formation of true erythrocytes.

C) Erythropoietin causes the erythrocyte colony-forming units to proliferate and mature. Erythropoietin acts primarily in later stages of erythropoiesis to induce the erythrocyte colony-forming units to proliferate and mature through the normoblast stage into reticulocytes and mature erythrocytes. It does not act directly on the stem cells, nor does it play a role in the sensation of hypoxia. Reticulocytes already lack a cell nucleus.

A nurse in a respiratory unit of a hospital is providing care for a client with end-stage lung disease. Consequently, measurement of the client's arterial blood gases indicates increased PCO2. Which of the following associated consequences would the nurse anticipate? A) A shift to the left of the oxygen-hemoglobin dissociation curve B) Lower than normal production of HCO3 C) Higher than normal production of H+ D) An absence of carbaminohemoglobin

C) Higher than normal production of H+ As a result of the combination of water and carbon dioxide, hydrogen ions are produced along with bicarbonate. This would be associated with a shift to the right of the oxygen-hemoglobin dissociation curve, increased bicarbonate output, and higher than normal levels of carbaminohemoglobin.

As a result of dehydration, a client's epithelial cells are producing insufficient amounts of mucus. Consequently, the client's mucociliary blanket is compromised. Which of the following changes would the care provider anticipate as a direct result of this change? A) Impaired function of the client's cilia B) Decreased levels of oxygen saturation C) Increased amounts of bacteria in the lungs D) Increased carbon dioxide levels

C) Increased amounts of bacteria in the lungs The primary role of the mucociliary blanket is to trap foreign particles and bacteria and thus prevent their entry into the lungs. Impaired ciliary function may result in an inadequate mucociliary blanket, but the opposite relationship is unlikely. Decreased oxygen and increased carbon dioxide levels may eventually result, but not as a direct or immediate consequence

A nurse practitioner is providing care for a client with low levels of the plasma protein gamma globulin. The nurse would recognize that the client is at risk of developing which of the following health problems? A) Jaundice B) Anemia C) Infections D) Blood clots

C) Infections The gamma globulins, antibodies of the immune system, protect against infectious diseases. Because alpha globulin transports bilirubin, a defect in this plasma protein might lead to jaundice; a defect of beta globulin, which transports iron, could be associated with anemia; and a defect of fibrinogen could result in blood clots.

A 44-year-old male hospital client with a diagnosis of end-stage acquired immunodeficiency syndrome (AIDS) has been placed on neutropenic precautions that limit his interaction with visitors, staff, and other clients. What is the underlying rationale for these precautions? A) His antibody-mediated immunity is compromised by his low production of neutrophils. B) Neutropenia limits the ability of his CD4 helper cells to present antigens. C) Insufficient levels of neutrophils make him particularly susceptible to infections. D) Cyclic neutropenia limits his body's ability to fight various infections.

C) Insufficient levels of neutrophils make him particularly susceptible to infections. A decrease in the number of neutrophils places an individual at risk for infection. Neutrophils are not directly involved in the antibody-mediated immune process, and his neutropenia is infection related, not cyclic.

A physician is explaining to a 40-year-old male patient the importance of completing his course of antibiotics for the treatment of tuberculosis. The physician explains the damage that could occur to lung tissue by Mycobacterium tuberculosis. Which of the following phenomena would underlie the physician's explanation? A) Tissue destruction results from neutrophil deactivation. B) Nonspecific macrophage activity leads to pulmonary tissue destruction and resulting hemoptysis. C) Macrophages form a capsule around the Mycobacterium tuberculosis bacteria, resulting in immune granulomas. D) Neutrophils are ineffective against the Mycobacterium tuberculosis antigens.

C) Macrophages form a capsule around the Mycobacterium tuberculosis bacteria, resulting in immune granulomas. Immune granulomas are caused by insoluble particles (Mycobacterium tuberculosis) that are capable of inciting a cell-mediated immune response. Neutrophils are deactivated, self-destructive, nor ineffective in cases of tuberculosis.

A 12-year-old boy has contracted a bacterial infection at school, and his body has responded by increasing leukocyte production. Place the following components of white blood cell production in the correct chronological order. Use all the options. A) Myeloblast B) Promyelocyte C) Myeloid stem cell D) Metamyelocyte E) Neutrophil

C) Myeloid stem cell A) Myeloblast B) Promyelocyte D) Metamyelocyte E) Neutrophil Granulocytes like neutrophils derive from the myeloid stem cells. The immature precursor cells for each of the cell lines are called blast cells. Myeloblasts, which are the granulocytic precursor cells, have round to oval nuclei, with delicate chromatin and a blue to gray cytoplasm. During the next stage of development, the myeloblasts are transformed into promyelocytes with similar nuclei, but with a cytoplasm containing many primary granules. In the subsequent metamyelocyte stage, the nuclei distort and become arclike, producing the band developmental stage. Metamyelocytes mature into neutrophils.

A group of nursing students were studying for their pathophysiology exam by quizzing each other about disorders of WBCs and lymphoid tissue. When asked what the first chromosomal abnormality that identified cancer was, one student correctly answered A) interleukin cells. B) BRCA-1. C) Philadelphia. D) PSA.

C) Philadelphia. One of the more studied translocations is the Philadelphia chromosome, which was the first chromosomal abnormality identified in cancer. Cytokines or chemical messengers, such as interleukin (IL)-1, IL-4, IL-6, and interferon, act synergistically to support the functions of the growth factors. BRCA is a genetic test for breast cancer testing. BRCA mutations place the female at risk for developing cancers of the breast and ovary; PSA stands for prostate-specific antigen. It is a protein produced by prostate cells and is utilized to screen for prostate cancer.

If an Rh-negative mother is giving birth to an Rh-positive infant, the nurse should be prepared to administer A) antihistamines like Benadryl. B) alpha interferon. C) Rh immune globulin. D) a monoclonal antibody like infliximab

C) Rh immune globulin. The injection of Rh immune globulin prevents sensitization in Rh-negative mothers who have given birth to Rh-positive infants if administered at 28 weeks' gestation and within 72 hours of delivery, abortion, or genetic amniocentesis. Antihistamines, alpha-interferon, or infliximab is not used in this situation.

A client who presented with shortness of breath and difficulty climbing stairs has been diagnosed with pulmonary fibrosis, a disease characterized by scarring of the alveoli. Upon assessment of the lungs, what clinical manifestations should the nurse expect? A) Rapid, deep breaths B) Wheezing throughout lung fields C) Short, shallow breaths D) Pursed-lip breaths with slow, steady breaths

C) Short, shallow breaths Scarring diminishes the elasticity of the lung tissue, resulting in noncompliant lungs that are more difficult to inflate. In order to maintain a sufficient tidal volume and oxygen level with the lungs that require extra work to expand, the individual must take shallower, more rapid breaths. The effort and time required for him to breathe deeply would detract from his ability to bring in enough air.

A child has been diagnosed with thalassemia. Which of the following comorbidities may occur as a result of having thalassemia? A) Hypocoagulation B) Iron deficiency C) Splenomegaly D) Neutropeniabb

C) Splenomegaly Thalassemia can result in enlargement of the spleen and liver due to increased hematopoiesis and red cell destruction. It is associated with thrombotic events, not hypocoagulation, as well as iron excess. Neutropenia is not associated with thalassemia.

An oncologist has ordered a bone marrow biopsy for a client and is explaining the reasons for the test and what the client might expect during the test. Which of the following explanations best reflects an aspect of a bone marrow biopsy? A) "I'll take a sample of your bone marrow from your breastbone or your spine." B) "I will be harvesting a sample of your stem cells for examination." C) "I need a more accurate count of your blood components than normal blood work is able to provide." D) "I need to get samples of the types of blood cells that your body is producing."

D) "I need to get samples of the types of blood cells that your body is producing." Bone marrow biopsy can be used to detect abnormal cell production and gain samples. A sample would be taken most likely from the iliac crest, and the biopsy would not focus on pluripotent stem cells. Bone marrow biopsy is not used for simple quantification of blood cellular components.

A community health nurse is conducting a class on the nutritional component for new mothers. Which of the following teaching points would be most justified? A) "Iron supplementation is not necessary provided you are breast-feeding your infant." B) "Be aware that cow's milk depletes your baby's supply of iron." C) "Your infant needs the same amount of iron as you but has far fewer sources for obtaining it." D) "If you choose to feed your baby with formula, ensure that it is iron fortified."

D) "If you choose to feed your baby with formula, ensure that it is iron fortified." Formula and cereals for infants should be iron fortified to preclude iron deficiency anemia. Breast-feeding does not necessarily mitigate the need for iron supplementation, and cow's milk does not deplete existing iron stores but fails to provide sufficient levels of absorbable iron. Infants and children have significantly higher iron needs than do adults.

Amniocentesis has suggested that a couple's first child will be born with sickle cell disease. The parents are unfamiliar with the health problem, and their caregiver is explaining the complexities. Which of the following statements by the parents would suggest a need for further teaching or clarification? A) "Our baby's red cells are prone to early destruction because of his or her weak membranes." B) "Not all of his or her red cells will be sickled, but low oxygen levels can cause them to become so." C) "Sickled cells can block his or her blood vessels, especially in the abdomen, chest, and bones." D) "Our son or daughter likely won't show the effects of sickling until he or she is school-aged because of the different hemoglobin in babies."

D) "Our son or daughter likely won't show the effects of sickling until he or she is school-aged because of the different hemoglobin in babies." Fetal hemoglobin in the infant is replaced by 8 or 10 weeks of age, and manifestations of sickle cell disease can begin at this time. Answer choices A, B, and C all convey the aspects of sickle cell disease.

A client with a diagnosis of hemolytic anemia has gone to a community-based laboratory for follow-up blood work. The lab technician confirms with the client that hematocrit is one of the components of the blood work. The client replies, "I thought the point of the blood work was to see how many red blood cells I have today." How could the technician best respond to the client's statement? A) "This result will tell your care provider about the number of red blood cells in a given quantity of your blood plasma." B) "Your hematocrit measures the average size of your red blood cells and indirectly measures your oxygen-carrying capacity." C) "The result will indicate how many of your red blood cells are new and young and will indicate your body's production rate of red cells." D) "The hematocrit measures the mass that your red blood cells account for in a quantity of your blood."

D) "The hematocrit measures the mass that your red blood cells account for in a quantity of your blood." Hematocrit measures the mass of erythrocytes in a given quantity of blood plasma. It does not measure the number of red cells, their size, or their production rate and age.

A 40-year-old male client is shocked to receive a diagnosis of mature B-cell lymphoma and is doing research on his diagnosis on the Internet. Which of the following statements that he reads on various Web sites is most reliable? A) "Like most forms of Hodgkin lymphoma, mature B-cell lymphoma often requires radiation treatment." B) "Doctors are able to diagnose mature B-cell lymphoma by the presence of Reed-Sternberg cells." C) "Unlike many other lymphomas, mature B-cell lymphoma is often self-limiting, and treatment is focused on symptoms." D) "The lymph nodes are usually affected, and often the spleen and bone marrow."

D) "The lymph nodes are usually affected, and often the spleen and bone marrow." The NHL subtypes of mature B-cell lymphoma may affect the lymph nodes, spleen, or bone marrow. It is not a type of Hodgkin lymphoma, and Reed-Sternberg cells would indicate Hodgkin lymphoma. It is not self-limiting and necessitates active treatment

A patient has been diagnosed with anemia. The physician suspects an immune hemolytic anemia and orders a Coombs test. The patient asks the nurse what this test will tell the doctor. The nurse replies, A) "They will wash your RBCs and then mix the cells with a reagent to see if they clump together." B) "They will look at your RBCs under a microscope to see if they have an irregular shape (poikilocytosis)." C) "They will be looking to see if you have enough ferritin in your blood." D) "They are looking for the presence of antibody or complement on the surface of the RBC."

D) "They are looking for the presence of antibody or complement on the surface of the RBC." The Coombs test is used to diagnose immune hemolytic anemias. It detects the presence of antibody or complement on the surface of the red cell. Answer choice A refers to direct antiglobulin test (DAT). Answer choice B refers to blood smear test. Answer choice C refers to iron stores test.

A 36-year-old woman with a diagnosis of antiphospholipid syndrome is receiving a scheduled checkup from her nurse practitioner. Which of the following teaching points would the nurse most likely prioritize? A) "It's important for you to do regular physical activity and maintain a healthy body weight." B) "Good nutrition and blood sugar control are important in your case." C) "You'll need to avoid taking nonsteroidal anti-inflammatory drugs when you have menstrual cramps." D) "You need to ensure your birth control pills don't contain estrogen."

D) "You need to ensure your birth control pills don't contain estrogen." Estrogen-containing birth control pills can predispose individuals with antiphospholipid syndrome to a thrombotic event. Exercise, nutrition, and blood sugar control are not particularly associated with management of antiphospholipid syndrome, and nonsteroidal anti-inflammatory drugs have no noted relevance to the disease.

A 5th grade elementary student asks the school nurse how much blood is in an entire body. The nurse should respond that the average grown-up adult has A) 2 to 4 cups of blood in his or her body. B) 3 pints of blood in total. C) 3 to 4 quarts of blood in his or her body. D) 5 to 6 L of blood throughout his or her body.

D) 5 to 6 L of blood throughout his or her body. The total volume of blood in the average adult is about 5 to 6 L, and it circulates throughout the body within the confines of the circulatory system.

A medical student is familiarizing herself with recent overnight admissions to an acute medical unit of a university hospital. Which of the following patients would the student recognize as least likely to have a diagnosis of antiphospholipid syndrome in his or her medical history? A) A 66-year-old obese male with left-sided hemiplegia secondary to a cerebrovascular accident B) A 90-year-old female resident of a long-term care facility who has been experiencing transient ischemic attacks C) A 30-year-old female with a diagnosis of left leg deep vein thrombosis and a pulmonary embolism D) A 21-year-old male with a diagnosis of cellulitis and suspected endocarditis secondary to intravenous drug use

D) A 21-year-old male with a diagnosis of cellulitis and suspected endocarditis secondary to intravenous drug use Stroke, transient ischemic attacks, deep vein thrombosis, and pulmonary emboli are all common manifestations of the hypercoagulability associated with antiphospholipid syndrome. Cellulitis, endocarditis, and other infectious processes are less likely to be correlated with antiphospholipid syndrome.

In which of the following patients, would diagnostic investigations least likely reveal increased thrombopoietin production? A) An 81-year-old woman with diagnoses of rheumatoid arthritis and failure to thrive B) A 55-year-old man with dehydration secondary to Crohn disease C) A 66-year-old woman with a diagnosis of lung cancer with bone metastases D) A 21-year-old woman awaiting bone marrow transplant for myelogenous leukemia

D) A 21-year-old woman awaiting bone marrow transplant for myelogenous leukemia Diseases such as myelogenous leukemia and other cases of primary thrombocytosis result in abnormalities in the thrombopoietin receptor and platelet binding. Cases of secondary thrombocytosis have an etiology rooted in increased thrombopoietin production. The common underlying causes of secondary thrombocytosis include tissue damage due to surgery, infection, cancer, and chronic inflammatory conditions such as rheumatoid arthritis and Crohn disease.

Which of the following individuals would most likely possess normal plasma cell synthesis and fully differentiated myeloid and lymphoid cells? A) A 7-year-old boy with a diagnosis of acute lymphocytic leukemia (ALL) B) A 70-year-old male who has acute myelogenous leukemia (AML) C) A 58-year-old female with HIV and multiple myeloma D) A 78-year-old male who has been diagnosed with chronic lymphocytic leukemia (CLL)

D) A 78-year-old male who has been diagnosed with chronic lymphocytic leukemia (CLL) Chronic leukemia is associated with proliferation of myeloid and lymphoid cells that are better differentiated than in cases of acute leukemia. Multiple myeloma would denote abnormalities of the plasma cells.

. A client with a gastrointestinal bleed secondary to alcohol abuse and a hemoglobin level of 5.8 g/dL has been ordered a transfusion of packed red blood cells. The client possesses type B antibodies but lacks type D antigens on his red cells. Transfusion of which of the following blood types would be least likely to produce a transfusion reaction? A) B- B) B+ C) A+ D) A-

D) A- The client's blood type is A- and would necessitate A- or O- donor blood. Other types would induce a transfusion reaction.

A 16-year-old female has been brought to her primary care physician by her mother due to the girl's persistent sore throat and malaise. Which of the following facts revealed in the girl's history and examination would lead the physician to rule out infectious mononucleosis? A) The girl has a temperature of 38.1°C (100.6°F) and has enlarged lymph nodes. B) Her liver and spleen are both enlarged. C) Blood work reveals an increased white blood cell count. D) Chest auscultation reveals crackles in her lower lung fields bilaterally.

D) Chest auscultation reveals crackles in her lower lung fields bilaterally. While fever, enlarged lymph nodes, splenomegaly, hepatomegaly, and leukocytosis would suggest mononucleosis, adventitious lung sounds are not associated with the disease.

Due to complications, a male postoperative patient has been unable to mobilize secretions for several days following surgery and develops atelectasis. Which of the following processes would his care team anticipate with relation to his health problem? A) Vasodilation in the alveolar vessels in the affected region of his lung B) Increased workload for the left side of the patient's heart C) Increased blood flow to the area of atelectasis D) Directing blood flow away from the lung regions that are hypoxic

D) Directing blood flow away from the lung regions that are hypoxic Regional hypoxia, such as with a diagnosis of atelectasis, is associated with vasoconstriction and redirection of blood away from, not toward, the affected area of the lung. This also contributes to an increased workload for the right side of the heart.

The blood work of a 44-year-old male patient with a diagnosis of liver disease secondary to alcohol abuse indicates low levels of albumin. Which of the following phenomena would a clinician be most justified in anticipating? A) Impaired immune function B) Acid-base imbalances C) Impaired thermoregulation D) Fluid imbalances

D) Fluid imbalances Albumin plays a central role in the maintenance of osmotic pressure and fluid balance. Immune function, acid-base balance, and thermoregulation are not functions of albumin

A female patient is requiring supplementary oxygen by face mask due to her reduced lung compliance. Which of the following pathophysiological processes is most likely a contributor to her low lung compliance? A) The woman's lungs have more recoil than a healthy person's. B) Her type II alveolar cells are producing a slight excess of surfactant. C) Turbulent airflow is taking place in the patient's large airways. D) Her thoracic cage is less flexible than when she was healthy.

D) Her thoracic cage is less flexible than when she was healthy. Impaired thoracic cage flexibility can be a contributor to reduced lung compliance. Increased recoil and a modest excess of surfactant would increase lung compliance, and turbulent flow in the airways is a normal, not pathological, finding.

Which of the following statements best conveys an aspect of the respiratory pressures that govern ventilation? A) Intrapleural pressure slightly exceeds that of the inflated lung. B) The chest wall exerts positive pressure on the lungs that contributes to expiration. C) The lungs are prevented from collapsing by constant positive intrapulmonary pressure. D) Negative intrapleural pressure holds the lungs against the chest wall.

D) Negative intrapleural pressure holds the lungs against the chest wall. Negative intrapleural pressure holds the lungs in place against the chest wall and prevents their natural elastic properties from causing them to collapse. Intrapleural pressure is negative in relation to the inflated lung, and the chest wall exerts negative pressure on the lungs that keeps them from contracting and contributes to inspiration. Intrapulmonary pressure oscillates between positive and negative relative to atmospheric pressure with expiration and inspiration.

A 53-year-old man presents with inability to concentrate, itching in his fingers and toes, elevated blood pressure, and unexplained weight loss. He is diagnosed with primary polycythemia. What will be the primary goal of his treatment? A) To control his hypertension B) To increase the amount of oxygen distributed by his red blood cells C) To reduce the mean size of his red cells D) To reduce the viscosity of his blood

D) To reduce the viscosity of his blood While hypertension may accompany polycythemia vera, the primary goal of treatment is to control the increase in blood viscosity that accompanies the disease. Polycythemia vera is not associated with increased corpuscular volume, and oxygen distribution is not a priority problem.

A new mother and father are upset that their 2-day-old infant is requiring phototherapy for hyperbilirubinemia. The pediatrician who has followed the infant since birth is explaining the multiplicity of factors that can contribute to high serum bilirubin levels in neonates. Which of the following factors would the physician be most likely to rule out as a contributor? A) The fact that the infant is being breast-fed B) Hypoxia C) Hepatic immaturity of the infant D) Transitioning of hemoglobin F (HbF) to hemoglobin A (HbA)

D) Transitioning of hemoglobin F (HbF) to hemoglobin A (HbA) Hemoglobin transition from HbF to HbA is not associated with hyperbilirubinemia. Breast-feeding, hypoxia, and immaturity of the young liver can contribute to hyperbilirubinemia.

The physician mentions the patient has developed alveolar dead space. The nurse recognizes that this means A) air that is moved in and out of the lungs with each breath. B) air that cannot participate in gas exchange and remains in the main bronchus. C) air is trapped in the conducting airways. D) alveoli are ventilated but not perfused.

D) alveoli are ventilated but not perfused. Alveolar dead space results from alveoli that are ventilated but not perfused.

The school nurse knows several children with hemophilia A. After recess, one student with hemophilia comes to the school nurse complaining of pain in his knee from falling on the playground. The nurse notes there is swelling in the knee and pain on palpation. The nurse should A) administer some NSAIDs to relieve the pain. B) wrap the knee in an ace bandage for compression. C) apply some warm compresses to the knee. D) notify parents to pick up the child and possibly administer factor VIII.

D) notify parents to pick up the child and possibly administer factor VIII. Prevention of trauma is important. ASA and other NSAIDs that affect platelet function should be avoided. Factor VIII replacement therapy administered at home has reduced the typical musculoskeletal damage. Wrapping with a bandage will not prevent damage. Warm compression will extend the bleed.

Which of the following patients would be most likely to be experiencing an increase in renal erythropoietin production? A) A 71-year-old smoker admitted to hospital with exacerbation of his chronic obstructive pulmonary disease (COPD) B) A 70-year-old woman admitted with dehydration secondary to an overdose of her potassium-wasting diuretic C) A 68-year-old man with a long-standing diagnosis of polycythemia vera D) A 21-year-old man with acute blood loss secondary to a motor vehicle accident 3 hours prior

A) A 71-year-old smoker admitted to hospital with exacerbation of his chronic obstructive pulmonary disease (COPD) Increases in erythropoietin production are associated with secondary polycythemia, and not polycythemia vera, a health problem that can be induced by the hypoxia resulting from smoking and lung disease. Dehydration is associated with relative polycythemia, and sudden blood loss would not manifest in increased erythropoietin production.


Conjuntos de estudio relacionados

DIFFERENCE BETWEEN TAXATION AND INTEREST RATES

View Set

3305 Client Assessment Lecture PrepU Mastery Level Questions Chapters 11, 16

View Set

PATH CH 24: Fluid and Electrolyte Homeostasis and Imbalances

View Set

Istorija, Murauskienė 02-27 konspektas 1 dalis

View Set

Chapter 5 Search Engine Marketing

View Set